Showing posts with label Con Law II. Show all posts
Showing posts with label Con Law II. Show all posts

Wednesday, April 30, 2008

Constitutional Law II

Dissents may become the majority years later.

Some rights are too important to be trusted to the people.

Emergency circumstances: Must shock the conscience

PROCEDURAL DUE PROCESS

§442 Procedural due process guarantees apply only to government action. There is no procedural due process issue if a private person or institution is depriving someone of liberty or property.

Has there been a deprivation? What level of intent is sufficient to demonstrate a deprivation under the due process clause?

§ 443 Procedural due process analysis applies only to an intentional deprivation of liberty or property

Daniels v. Williams (1986)

The Court rejected a 42 USC § 1983 action by an inmate who sued a jail officer for injuries suffered when the inmate slipped and fell on a pillow negligently left on a stairway by the jail officer. The Due Process Clause is not implicated by a state official’s negligent act causing unintended loss of injury of life, liberty, or property.

Davidson v. Cannon (1986 – decided same day as Daniels)

A prisoner claimed prison authorities violated his due process rights by failing to protect him from attack by another prisoner. The Court held that the allegation of government negligence was insufficient to state a claim under the Due Process Clause.

County of Sacramento v. Lewis (1998)

Issue: Whether a police officer violates the Fourteenth Amendment’s guarantee of substantive due process by causing death through deliberate or reckless indifference to life in a high-speed automobile chase aimed at apprehending a suspected offender.

Holding: No. In such circumstances only a purpose to cause harm unrelated to the legitimate object of arrest will satisfy the element of arbitrary conduct shocking the conscience, necessary for a due process violation. High-speed chases with no intent to harm suspects physically or to worsen their legal plight do not give rise to liability under the Fourteenth Amendment, redressable by an action under § 1983. The fault claimed in this case fails to meet the shocks-the-conscience test.

Reasoning: The due process guarantee does not entail a body of constitutional law imposing liability whenever someone cloaked with state authority causes harm. Like prison officials facing a riot, the police on occasion calling for fast action have obligations that tend to tug against each other. A police officer must balance on one hand the need to stop a suspect and show that flight from the law is no way to freedom, and, on the other, the high speed threat to everyone within stopping range, be they suspects, their passengers, other drivers, or bystanders.

When the government’s failure to protect a person from privately inflicted harm a deprivation under the due process clause?

§363 A state’s failure to protect an individual from violence by a private party does not constitute a violation of the Due Process Clause. The government has no obligations to protect a person from harm by another person. There will be state action, however, if a person is injured while under the custody of control of the state.

The Court found no state action when a state social worker investigated allegations of child abuse, found no corroborating evidence, but left the little boy in his home where he was severely beaten by his father. The boy suffered brain damage and, as a result of the beatings, became profoundly retarded. The Court rejected the argument that the social worker’s failure to act deprived the boy of due process. A state’s failure to protect a person from private violence does not involve state action unless the state takes a person into custody and holds that person there involuntarily.

Class note re: when the government limits the ability of a person to protect himself or when the government took an affirmative step to place the injured person in danger

DeShaney v. Winnebago County (1989) (p. 1011 text)

Nothing in the language of the Due Process Clause itself requires the State to protect life, liberty, and property of its citizens against invasion by private actors. The purpose of the DPC was to protect the people from the State, not to ensure that the State protected them from each other. If the DPC does not require the State to provide its citizens with particular protective services, it follows that the State cannot be held liable und the Clause for injuries that could have been averted had it chosen to provide them. As a general matter, then, we conclude that a State’s failure to protect an individual against private violence simply does not constitute a violation of the DPC.

The affirmative duty to protect arises not from the State’s knowledge of the individual’s predicament or from its expressions of intent to help him, but from the limitation which it has imposed on his freedom to act on his own behalf.

Brennan’s Dissent:

The State’s knowledge of an individual’s predicament and its expressions of intent to help him can amount to a limitation on his freedom to act on his own behalf or to obtain help from others.

Is life, liberty, or property deprived?

What is property? Two approaches:

§465 Welfare recipients are entitled to a pre-termination hearing.

Goldberg v. Kelley (1970)

The Court ruled that welfare recipients are entitled to an evidentiary hearing prior to termination of their benefits. Welfare benefits are needed for the purposes of subsistence, so the Court refused to allow a summary termination of benefits pending resolution of a recipient’s eligibility.

§462 No express or implied tenure rights in a teaching position at a public university means no property interest.

Board of Regents v. Roth (1972)

The Court found that a state university professor had no property interest in his teaching job. The professor had been hired for one year, and there were no express or implied tenure rights granted to him. He had no entitlement to his job, simply a unilateral expectation of continued employment. Since he had no property interest in his job, he was not entitled to a hearing, or even a formal statement of the reasons why he was not retained.

The Court generally has adopted the latter approach (Goldberg) to defining property as an entitlement — that is, as a reasonable expectation to continued receipt of a benefit (as opposed to a mere expectation).

Goldberg is the key case. It recognizes welfare benefits as property. Roth is entitlement – a continued expectation of the receipt of the benefit. It doesn’t matter under Roth how important it is. Under Goldberg all that matters is, do you think it is important?

Is life, liberty, or property deprived?

Deprivations of “Liberty.” What is liberty?

Is harm to reputation a deprivation of liberty?

§468 No formal hearing is required before a student receives a temporary disciplinary suspension.

Goss v. Lopez (1975)

The Court ruled that, as a general matter, before a student is given a temporary disciplinary suspension (ten days or less) from a public school, he or she must be given notice (oral or written) of the reasons for the suspension, and at least an informal opportunity to respond to the charges. A full-blown adversary hearing is not required.

§459 Injury to reputation alone is not a liberty interest protected under procedural due process; however, if injury to reputation is linked to some other interest, such as pursuing employment, there may be a liberty interest involved.

Paul v. Davis (1976)

The Court rejected a due process argument by a plaintiff whose reputation was allegedly injured by police circulation to merchants of a flyer designating plaintiff an “active shoplifter.” Since damage to his reputation was not tied to any other interest such as employment, there was not protectable liberty interest.

When do prisoners have a liberty interest?

Sandin v. Conner (1995) (1034 text)

A prison inmate who was strip searched became angry and retorted with foul language against the officer. When charged with disciplinary infractions, the committee refused Conner’s request to present witnesses at the hearing. The Court held that the State’s actions in placing him in 30 days segregation did not work a major disruption in his environment.

Wilkinson v. Austin (2005) (96 supp)

Inmates who had been moved from a state penitentiary to a maximum security prison brought a class action suit and claimed that the state’s procedures for moving inmates to the maximum security prison violated the due process requirements of the Fourteenth Amendment to the US Constitution. The Court held that the state created the inmates’ liberty interest in avoiding movement to the maximum security prison by creating and implementing policies for determining which prisoners should be moved to the maximum security facility. The Court also held that the state’s procedures for moving inmates sufficiently protected the inmate’s liberty interest in avoiding erroneous transfer and the state’s interest in running its prison system efficiently. Thus, the state’s procedures for moving inmates to its maximum security prison did not violate the inmate’s due process rights.

Is life, liberty or property deprived?

Deprivations of “Life” - What is Life?

Life is easier to define than liberty or property - Issue sometimes tangentially raised in controversy over abortion or euthanasia though

Usually, claims concerning state deprivations of life are litigated under constitutional provisions other than due process.

E.g., challenges to the constitutionality of the death penalty usually involve the Eighth Amendment

What Procedures are Required?

§466 Disability benefit recipients are not entitled to a pre-termination hearing.

Mathews v. Eldridge (1976)

The Court ruled that there is no right to a hearing prior to the termination of disability benefits. The Court said that the receipt of disability payments is simply not as crucial as receipt of subsistence welfare payments.

Balance 3 factors:

First, the private interest that will be affected by the official action;

Second, the risk of an erroneous deprivation of such interest through the procedures used, -- and the probable value, if any, of additional or substitute procedural safeguards;

Third, the Government’s interest, including the function involved and the fiscal and administrative burdens that the additional or substitute procedural requirement would entail.

Procedural due process analysis involves three general issues:

1. Has there been a “deprivation”?

2. Is it a loss of “life, liberty, or property”?

3. What procedures are required?

Analytical Strategy – procedural due process

Question 1: Is there a life, liberty, or property interest?

If no, then no procedural due process implications

If yes, go to Question 2

Question 2: Has there been a deprivation of that interest?

If no, then no procedural due process required

If yes, go to Question 3

Question 3: What process is required?

Pre-deprivation or post-deprivation process

Formality of process

Full evidentiary hearing versus something less

Test: Mathews v. Eldridge balancing test

Nature of the individual interest

Risk of deprivation and benefit of additional procedures (courts tend to be deferential)

Nature of state interest

SUBSTANTIVE DUE PROCESS

The difference between procedural and substantive due process

Procedural due process focuses on the procedures that government must follow when it takes away a person’s life, liberty, or property.

Substantive due process focuses on whether the government has an adequate reason – a sufficient substantive justification – for taking away a person’s life, liberty, or property.

Under substantive due process analysis, the Court reviews the substance of legislation to see if it unduly interferes with an individual’s rights or interests that are protected by the Due Process Clauses of the Constitution.

Substantive Due Process

Used in two major ways in American History:

Protection of Economic Rights

Protection of Fundamental Rights

US v Carolene Products (1938) (p. 543)

Introduction to levels of scrutiny court applies in substantive due process cases

§482

The Court upheld a federal law that prohibited the shipment of “filled milk” in the interstate commerce (“filled milk” refers to milk or cream to which fat or oil has been added). The Court said it should uphold the law if any state of facts existed, or could have been presumed to exist, which would support the law. Any arguably rational basis for the law would be enough to uphold it.

Fifth Amendment: No person shall be ... deprived of life, liberty, or property, without due process of law...

Fourteenth Amendment: No State shall ... deprive any person of life, liberty, or property, without due process of law...

When we are talking about somebody’s individual liberty and trying to identify some individual liberty that deserves protection – in other words, the government has acted and we are trying to find something that is going to limit or protect that government action – we need to find someplace to hang that right (coat hanger). The right for parents to control the upbringing of their kids we are going to hang on the Fourteenth Amendment – the Due Process Clause or maybe even the Equal Protection Clause.

The Big Picture

As you go through, and you see something the government has done that is regulating some freedom – speech for example. I can hang my right on the First Amendment. You’ve got to find somewhere in the Constitution to hang that right in order for you to claim that there is a limit on the government power and how the government power was enacted.

Review: The Due Process Clauses

The Fifth and Fourteenth Amendments, respectively, provide that neither the United States nor state governments shall deprive any person “of life, liberty, or property without due process of law.”

Review of the difference between procedural and substantive due process

Procedural due process focuses on the procedures that government must follow when it takes away a person’s life, liberty, or property.

Substantive due process focuses on whether the government has an adequate reason - a sufficient substantive justification - for taking away a person’s life, liberty, or property.

Substantive Due Process Analysis:

The Court looks at the substance of the law to see if it interferes with an individual’s right or interest to free speech, contraception, etc.

KEY POINTS

Main concept: Certain rights – what are sometimes called fundamental rights – are so important that courts will closely scrutinize any attempt by the government to substantially impair their exercise.

Fundamental rights may be textual (found in Constitution or Bill of Rights) or non- textual

Our concern, at this point, is with non-textual fundamental rights

If the government passes a law regulating commerce or economic rights (not so important nowadays) the Court will not look at it very close (pair of glasses) and use a rational basis – does the government have a legitimate interest in regulating this economy? And are the means they are using rationally related to it? Burden of proof is on claimant.

If the government passes a law regulating a fundamental interest, the Court will look at it very closely (microscope) and use strict scrutiny. Does the government have a compelling interest in interfering with my right to control the upbringing of my child, religious conscience, etc. The government also has to have narrowly tailored means (least restrictive means) to accomplish their very important compelling purpose. Burden of proof is on the government.

Why does it matter? Because the government’s choice to say something is a fundamental right versus an unimportant right will determine the likely outcome of the case. If the Court looks at it as a fundamental right and requires the government to show a compelling interest and a least restrictive means, the chances are the law will be struck down.

On the other hand, on a low-level economic interest, the Court will uphold almost all of those laws.

IMPORTANT:

Step 1:

Once you identify the right or interest that is abridged, you must argue that it is some aspect of the liberty interest under the appropriate DPC. You have to hang it on some right and most of the time we are hanging it on the Fourteenth or Fifth Amendment of the DPC.

Step 2:

Then you have to determine if it is a low-level liberty interest or a fundamental (compelling) liberty interest.

Step 3:

Apply the level of scrutiny.

Originalist

Fundamental rights are found in the Bill of Rights. But what about the right to infringe on the upbringing of a child? That’s not found in the Bill of Rights. An originalist may not necessarily like the fact that the Court is finding that law in the Constitution because it is not expressly there even though that is a politically conservative issue.

Active Liberty

On the other hand, there are some politically/jurisprudentially liberal decisions (judicial activism), like Roe v. Wade. Be cognizant of the approaches of the justices.

My goal is that by the end of the course you will have a reason as to why you are an originalist or why you are a judicial activist.

Final Exam: The essay will be some type of hot-button case. Politics aside, how would this case be decided by an originalist? How would it be decided by a non-originalist? There are cases that will come before the Court in the next 5 years and we don’t know what the answer is to those questions. The more that you can understand the different positions on how to get to point A to point B, you will understand Constitutional law. Example: gay marriage.

If the creates a classification of people and treats one group different from another group of people, apply Equal Protection analysis.

If the law applies to everybody the same, apply Substantive Due Process analysis.

For this week’s SDP cases: Was the Court saying something was a fundamental right?

The Ninth Amendment –

“The enumeration in the Constitution of certain rights, shall not be construed to disparage others retained by the people.”

U.S. Const. Ninth Amendment

Textual justification for protecting non-textual fundamental rights -- or fig leaf?

The author says that the Ninth Amendment gives judges the authority to find these other rights in the Constitution (others retained by the people). How would an originalist respond to that? That Congress should decide and that the Ninth Amendment is really only a “fig leaf.”

BASIC ANALYTICAL STRUCTURE/ATTACK STRATEGY


Basic due process analysis involves 4 questions

First, is there a fundamental right?

Second, is the right infringed?

Third, is the government’s action justified by a sufficient purpose?

And fourth, are the means sufficiently related to the goal sought?

If no fundamental right, then go to the rational relation test.

The Battle

The justices are at war with each other – not to the answer of the question but how to answer the question. Do I say a fundamental right is something I see right in the Constitution or can I go someplace else in the penumbras that emanate from those different rights to find it? Look for that ongoing battle in these decisions and you will be master of constitutional jurisprudence.

Identifying NON-TEXTUAL FUNDAMENTAL RIGHTS

Did you find a way to discern that a fundamental right is not in the Constitution’s text?

Note originalism/non-originalism debate

How are non-textual rights identified now? Examples:

Is the asserted right implicit in the concept of ordered liberty?

Is the asserted right deeply rooted in our nation’s history or traditions?

Question 1: Is there a fundamental right? Should the Court find fundamental rights not supported by the Constitution’s text or framer’s intent? If no, rational basis test.

Question 2: If yes, is the right being substantially impaired? If no, rational basis test applied. If yes, examine government interest.

Question 3: If yes, can government show a compelling interest to justify the impairment? If no, then it is a due process violation.

Question 4: If yes, are the means necessary or narrowly tailored to achieve that interest? Is the government using the least restrictive means to accomplish the compelling interest? If no, then there is a due process violation. If yes, then no due process violation.

What is a compelling interest? Homeland Security, War on Terror, etc. What about mandatory vaccinations against STD’s? Is the government using the least restrictive means to accomplish that? Pa is going to assert his fundamental right to raise his kids and strike the statute down. The government responds by saying, we agree it is a fundamental right but we have a compelling interest – the health and welfare of our kids. The next question is, is the government using the least restrictive means to protect the health and safety of our kids. If the government cannot show least restrictive means then Pa wins.

Abortion uses the Unduly Burdened Test

For Exam: There are no model answers in the Exam Bank. The law changes too much on this subject. The model answers are the rationales of the cases we read. Write an answer and see if it matches what the case says.

Introduction to Fundamental Rights Under Due Process (and Equal Protection)

Some liberties so important they are deemed fundamental rights à Generally the government can’t infringe them unless strict scrutiny is met e.g.:

Rights protecting family autonomy;

Procreation;

Sexual activity;

Medical care decision making;

Travel; voting; and access to the courts;

Freedom of Speech, Religion, Criminal Procedure

For Exam – Strategy for fundamental rights: When something is happening to a person (state action/law) and that person wants to raise an individual right, that person has to go to some other part of the Constitution to hang the right (Bill of Rights, Fourteenth Amendment liberty interest, First Amendment free speech, etc.). If you can’t place it in the Constitution, then you can’t go to the government and say you have a way to limit what they are doing.

CONSTITUTIONAL PROTECTION FOR FAMILY AUTONOMY

What rights of family autonomy are protected?

The Right to Marry

The Right to Custody of One’s Children

The Right to keep the Family Together

The Right of Parents to Control upbringing of their Children

The Right to Marry – Fourteenth Amendment liberty interest

Loving v. Virginia (1967) (p. 821 text) §588

Boddie v. Connecticut (1971) (826 text)

Zablocki v. Redhail (1978) (p. 822 text). S

* EP case; concur uses SDP

Loving v. Virginia

The Court invalidated a Virginia law which criminalized marriage between a white and a non-white person. Any non-white racial classes could intermarry with no criminal sanctions. The Court thoroughly rejected the argument that equal protection guarantees were not violated because whites and blacks were treated equally – neither could marry the other. It recognized this law for what it was – a blatant attempt by the state to reinforce invidious stereotypes of racial inferiority (one of the asserted state interests underlying the law was to prevent “a mongrel breed of citizens”). Be aware that the law also abridged the fundamental right to marry.

Wagner: If you can’t get married, you can’t make babies and continue the nation. If the government passes a statute that infringes on the right to marry, it will be looked at under strict scrutiny.

[§534] Waiver of Court Fees to Get a Divorce. The Court held that a state must waive court fees for an indigent seeking a divorce.

Boddie v. Connecticut

The Court invalidated the application of a state’s court fees and costs to an indigent seeking to go to court to get a divorce. Since this was the only forum in which a divorce could be obtained, a state could not use a system which excluded poor people from that forum. Does Boddie mean that there is a fundamental right to divorce?

Wagner: You hang this right on the Fourteenth Amendment. I have a liberty interest to get married, and if I can’t get divorced I can’t get married.

§505 Right to Marry. The Court has held that the right to marry is an aspect of liberty protected by the Due Process Clauses of the Constitution.

Zablocki v. Redhail

The Court invalidated, under equal protection, a Wisconsin law that refused to grant a marriage license to anyone who had minor children whom he or she was obligated to support, unless there was proof of payment of the child support obligations. The classification effected by the law significantly interfered with the exercise of the fundamental right to marry.

Wagner: How does the Court determine that the right is so important that we are going to give it strict scrutiny if the government passes a law infringing on the right? What’s the states compelling interest? The protection of kids. If this wasn’t a fundamental right, we apply the rationally related to a government purpose standard. The underlying battle is whether it’s appropriate for the Court to create that fundamental right on their own or whether that is something the legislators should do.

*The fact that you are married and have to pay higher taxes is not a substantial impairment.


What rights of family autonomy are protected?

The Right to Custody of One’s Children

Stanley v. Illinois (1972) (p. 827 text)

Compare: Michael H. v. Gerald D (1989) (829 text).

§654 States may not conclusively presume that all unmarried fathers are unfit parents.

Stanley v. Illinois

The Court invalidated an Illinois statue which conclusively presumed that all unmarried fathers are unfit parents. Upon the mother’s death, the State, rather than the unmarried father, would get custody of a child. The Court said this classification was clearly overbroad in that it disadvantaged too many unmarried fathers (those who would be fit parents), and the state must provide individualized hearings on the question of fitness.

§660 A state may presume that a child born to a married woman is a child of her marriage.

Michael H. v. Gerald D

The Court upheld a California law which conclusively presumed that a child born to a married woman is a child of the marriage. Michael was, in all probability, the natural father of a child to whom he asserted parental rights. However, Gerald, not Michael, was the husband of the child’s mother. Under California law, Gerald was conclusively presumed to be the child’s father. In a plurality opinion, Justice Scalia rejected Michael’s argument that the law created an unconstitutional irrebuttable presumption. Justice Scalia reasoned that since Michael had no fundamental parental interest simply because of genetic parenthood, the state’s interest in protecting marital relationships was sufficient to override Michael’s low-level liberty interest in seeing his child. Justice Brennan dissented, arguing that such a conclusive presumption has the fatal procedural flaw of not providing a hearing in which the adulterous father can offer proof of paternity.

The Right to keep the Family Together

Moore v. East Cleveland (1977) ( 835 text)

Limits on Moore:

Individuals must be related to be family

The Supreme Court refuses to find infringement of right to keep family together unless there is a direct and substantial interference

§519 Family Living Arrangements are protected as an aspect of liberty, but the Court has not clearly elevated this activity to fundamental right status.

Moore v. East Cleveland

The Court invalidated a zoning ordinance which prohibited Grandma from living with her two grandsons, who were cousins, but would have allowed the living arrangement if the grandsons were brothers. While not crystal clear on the level of scrutiny it used, the Court said it would “examine carefully” any zoning law which affected freedom of personal choice in matters of family life.

Wagner: Cousins are blood and that’s part of the liberty interest that the DPC includes and that means that if the government is going to infringe on your Fourteenth Amendment right, they’ve got to give a substantive justification and it has be a compelling one. The government has to show that the means they are using to accomplish that compelling interest are the least restrictive.

The Right of Parents to Control upbringing of their Children

Meyer v. Nebraska (1923) (p. 839 text)

Pierce v. Society of Sisters (1925) (840 text)

Wisconsin v. Yoder, (841 text)

Troxel v. Granville (2000) (p. 842 text)

§495 Parents have the right to determine which schools their children will attend.

Meyer v. Nebraska

The Court invalidated a state law which prohibited the teaching of any foreign language (other than the “dead” ones) in any elementary school. The Court relied upon the unenumerated rights to marry, establish a home, and bring up children.

Pierce v. Society of Sisters

The Court invalidated a state law which required all students to attend public, rather than private or religious, schools. The Court referred to the right of parents and guardians to direct the upbringing and education of children under their control.

§980 Mandatory Education Laws. A state cannot apply a mandatory education law to the Old Order Amish.

Wisconsin v. Yoder

The Court upheld a state court’s reversal of the convictions of members of the Old Order Amish community who refused to send their children to state-certified schools through the age of sixteen. The Amish children did go to school through the eighth grade. The Court applied strict scrutiny, asserting that the Wisconsin compulsory education law violated the Free Exercise rights of the Amish parents, as well as their unenumerated right to direct the educational upbringing of their children.

§497 A state may not grant broad-based non-parental visitation rights only upon a showing that the visitation would be in the best interest of the child.

Troxel v. Granville

The Court invalidated a Washington statute that permits any person to petition for visitation rights at any time and authorizes a state court to grant visitation if it decides it would be in the child’s best interest. Writing for a plurality, Justice O’Connor reaffirmed that parents have a fundamental right to make decisions concerning the care, custody, and control of their children. The statute could be used even if there were no finding of unfitness on the part of the parents, and the parents’ decision regarding the best interest of the child received no weight from the Court. The plurality however, did not employ any particular level of judicial scrutiny. Justice Thomas, concurring, agreed that there is a fundamental right of parents to rear children and argued that any abridgement of that right should receive strict scrutiny.

Wagner: As ma’s attorney you argue that the government cannot meet strict scrutiny. The state has to show a compelling interest for the law. The state hasn’t narrowly tailored the law.

Scalia sees a Ninth Amendment right of parents to rear their children in the Declaration of Independence where it says, Rights endowed by our Creator, and this is one of those rights. Scalia dissents in this case and says the legislature should decide this.

Ginsberg sees a Ninth Amendment right; therefore, I declare it to be a right.

CONSTITUTIONAL PROTECTION FOR REPRODUCTIVE AUTONOMY

What rights related to reproduction are protected?

the right to procreate;

the right to purchase and use contraceptives;

the right to abortion.

1. The right to procreate

Buck v. Bell (p. 847 text), (1927)

Skinner v. Oklahoma, (1942) (p. 849 text)

Note: this is an EP case

Buck v. Bell

The Circuit Court of Amherst County, Virginia ordered the superintendent of the State Colony for Epileptics and Feeble Minded to perform the operation of salpingectomy (removal of fallopian tubes) upon Carrie Buck for the purpose of making her sterile. The state interest was to prevent more imbeciles as three generations of imbeciles was enough.

§504 The Court has recognized marriage and procreation as fundamental rights that are protected by the Due Process Clauses of the Constitution.

Skinner v. Oklahoma

Text: The Supreme Court never overruled Buck v. Bell or repudiated Justice Holmes’s opinion, but Skinner v. Oklahoma 15 years later implicitly does so by recognizing a fundamental right to procreate. During the time between the two cases, the eugenics movement waned, certainly in large part to seeing its implementation in Nazi Germany.

The Court invalidated, under equal protection, an Oklahoma law that authorized the sterilization of repeat felons whose crimes involved moral turpitude. The law was designed to cleanse the gene pool of the offspring of recidivist felons. Under the law, however, white-collar crooks were exempted from the sterilization requirement. The Court used strict scrutiny because the law deprived certain persons of the fundamental rights of marriage and procreation. This case involves a non-suspect classification (one set of recidivist felons versus another) that in turn abridged fundamental rights. If the legislature had said that all recidivist morally turpitudinous felons were to be sterilized, this would be a substantive due process case.

Wagner: Now we have a fundamental right to procreate which is found in the Fourteenth Amendment.

2. The right to purchase and use contraceptives

Griswold v. Connecticut (1965) (p. 850 text)

Emanations and Penumbras

Eisenstadt v. Baird (1972) (p. 856 text)

EP case

Carey v. Population Services (1977)(p. 857 text).

§506 The Court has recognized the right of privacy as encompassing the right of married couples to receive information about contraceptives.

Griswold v. Connecticut

The Court invalidated a law which made it a crime to give advice or information about contraceptives to married couples. The Court said that the law violated a fundamental right to marital privacy. Justice Douglas said the right to marital privacy came from penumbras (shadows), formed by emanations from the First, Third, Fourth, Fifth, and Ninth Amendments (enumerated rights have certain corollary rights which flow from them). Justice Douglas did not place the right to marital privacy in the liberty provision of the DPC, but that is where the Court places it today.

Wagner: Would an originalist do that? If we have a right of privacy in the Constitution, then we have a right to purchase contraceptives. A functionalist or judicial activist would say this is a good way of getting to this right. Justice Douglas finds a right of marital privacy and applies it to a statute that says you can’t sell contraceptives. It’s not based upon reproductive choice, it’s based upon keeping the police from looking into my window.

§507 The Court has held that unmarried persons, as well as married couples, have a constitutional right to access to contraceptives.

Eisenstadt v. Baird

The Court invalidated, under equal protection, a Massachusetts law criminalizing the distribution of contraceptives to unmarried persons. This law violated the rights of single persons, under the Equal Protection Clause, to be treated the same as married persons in terms of access to contraceptives. In effect, the Court recognized a fundamental right of unmarried adults to have access to contraceptives.

Wagner: The right of privacy moves beyond Griswold

§508 The Court has held that minors have a privacy right to obtain non-prescription contraceptives.

Carey v. Population Services Int’l.

The Court invalidated a New York law which, in part, prohibited the sale of non-prescription contraceptives to persons under the age of 16. A plurality said that a state law inhibiting the privacy rights of minors would be valid only if it advanced some significant (not compelling) state interest. A ban on contraceptives to minors would not serve the state interest of deterring underage sex.

3. The right to abortion

Roe v. Wade (1973) (p. 859 text)

Webster v. Reproductive Health Services (1989)(p. 865-66)

Planned Parenthood v. Casey (1992) (867 text) - Current test in abortion cases

§509 The Court has held that a woman has a fundamental right to decide whether or not to terminate her pregnancy.

Roe v. Wade

The Court invalidated state laws criminalizing abortions, except in a case where the woman’s life was in danger. The Court, 7-2, ruled that a pregnant woman has a fundamental right to decide whether or not to terminate her pregnancy. The Court recognized two state interests, each of which becomes compelling at a different point during pregnancy. A state’s interest in maternal health or life becomes compelling at the end of the first trimester, providing a basis for regulation during the second and third trimester. A state’s interest in potential human life becomes compelling at viability (24-28 weeks) and provides a basis for a state restricting or prohibiting abortion during the third trimester. The Court also held that a fetus is not a person entitled to protection under the Fourteenth Amendment.

Wagner: Underlying the interests, there is a huge battle of who should be making this decision – should it be the Court or should it be the legislature? If the state can define the fetus as a life it then has the right to protect it and not deprive it without due process. The Court could have said that life begins at conception; therefore, the fetus is a person. If you had a law that permitted abortion, it would be unconstitutional as it would prevent the baby from having fundamental rights or equal protection under the law. Abortion would be murder. Smoking and drinking while pregnant would be child abuse. On the upside – tax laws, car pool lanes, etc. The originalist argument is that the Court could have left it up to the state legislatures to decide however they wanted to do it – let the people rule, not the courts. The criticism of that is that this is a right too important for the people to decide. The Court also could have said that restrictive abortion laws deny women equal protection under the law. The Court could have also said the state does not have a compelling interest justifying restrictions on abortion unless the fetus is viable. And if the fetus is viable, then at that point the state has an interest in protecting life – that’s the approach Roe took. In the second trimester the state has a compelling interest in protecting the mother. In the third trimester the state has an interest in protecting life.

§516 Justice O’Connor has adopted an “undue burden” analysis in abortion cases. Post-viability, a state may restrict or prohibit abortion. Pre-viability, a state may not pass a law which, by purpose or effect, imposes an undue burden on a woman’s decision to terminate her pregnancy.

Planned Parenthood v. Casey

In Casey, the Court reviewed five provisions of a Pennsylvania law regulating abortion. The Court upheld an informed consent requirement for adult women, a requirement that an adult woman wait twenty-four hours after receiving information before she can have an abortion, a requirement that a minor get the informed consent of one of her parents before she has an abortion (a judicial bypass was provided if the minor does not want to or cannot get the consent of a parent), and certain reporting requirements. The Court struck down a spousal notification requirement.

Although the Court retained the central holding of Roe, that a woman has a fundamental right to decide whether to terminate her pregnancy, seven justices rejected the trimester framework of Roe. Four justices (Rehnquist, White, Scalia, and Thomas) voted to overrule Roe completely. Two justices (Blackmun and Stevens) voted to retain all of Roe. Three justices (O’Connor, Kennedy, and Souter) voted to throw out the trimester framework, but retain the central right to decide. Justice O’Connor, writing the operative opinion split pregnancy into two parts: pre-viability and post-viability. Post-viability, a state may seriously restrict, even prohibit abortion. Pre-viability, a state may impose relatively non-intrusive requirements, but may not place an “undue burden” on the woman’s decision to abort. An undue burden exists when a state law has the purpose and effect of placing a substantial obstacle in the path of a woman who wants to abort a non-viable fetus.

Criminalizing abortion would be an undue burden as would spousal consent, or the spousal notice struck down by Casey. Some informed consent laws, and a twenty-four waiting period for an adult woman would not be undue burdens. Many of the restrictions received by the Court in pre-Casey cases will have to be revisited to see how they fare under the undue burden standard of Casey.

Wagner: It’s a woman’s body and she should be able to control it at all stages. The implications of that are that the government cannot stop a person from having an abortion but the state could regulate perhaps the way that the abortion occurs or if the fetus was alive after the abortion it would make it an award of the state.

Text: Government Regulations of Abortions. After Casey, the government can regulate abortions performed prior to viability so long as there is not an undue burden on access to abortions.

Constitutional Law II – Week 3

Wagner: We’ve seen the government passing statutes that infringe upon liberty rights. The question becomes, if that is a fundamental right, does the government have a compelling reason, and can they give a sufficient compelling justification on why they are infringing upon it?

If they did have a compelling reason, are they using narrowly tailored means in the statute and are they using the least restrictive means to accomplish that compelling objective when they are infringing upon somebody’s rights, liberty, or property?

When the government passes a law that regulates or prohibits abortion, the government is asserting two interests:

1. Protecting the life of the mother; and

2. Protecting human life.

In Roe you have a trimester interest:

End of First trimester: State’s compelling interest in maternal health.

Second trimester: State’s compelling interest in potential human life begins at viability (24-28 weeks).

Third trimester: State’s restricting or prohibiting abortion.

The Court held that a fetus is not a person entitled to protection under the Fourteenth Amendment.

Modern Substantive Due Process & the Protection of Fundamental Rights

Planned Parenthood v. Casey (1992) (867 text) - Current test in abortion cases

No longer is strict scrutiny used in evaluating government regulation of abortion.

This is the complaint of Blackmun and Stevens in dissent.

New test is whether government regulation places an “undue burden” on a woman’s right to abortion.

State may regulate abortion before viability so long as does not place undue burden on the right.

Wagner: Casey changes the framework of Roe. The court is trying to deal with science in some sense. Since Roe, abortions are becoming safer for the life of the mother earlier. At the same time, technology regarding viability is earlier. The trimester framework does not work any more because of science on both sides of the argument. The Court discards trimester framework and instead replaces it with the undue burden test.

An undue burden: Does a state regulation have a purpose or effect of placing a substantial obstacle in the path of a woman seeking a non-violent means?

The government can regulate prior to viability as long as there is no undue burden to the right to an abortion. The Court’s conclusion (profound interest) on why they can regulate abortion is to preserve the life of the unborn. Throughout the pregnancy the state can take measures to make sure that the mother’s choice is an informed choice, and that those measures are not going to be struck down under the undue burden test as long as the purpose is to persuade the mother to choose life over abortion.

If the government is passing measures to advance that interest of protecting the unborn potential life, interest is not going to be struck down or invalidated by the Court as long as the reason that the government is doing that is to persuade the mother to choose childbirth over abortion.

The Court did not overrule Roe, but it sounds like they are choosing sides. Would another regulation survive?

The difference from Roe: The joint opinion specifically said that they are promoting a profound interest in potential life throughout the pregnancy. They are giving life value pre-viability as well as post-viability. We are still balancing against an undue burden because we still have a right to an abortion. Casey is saying, if we read Roe correctly, Roe was using a profound interest in protecting life and therefore any measure that the government is using that is protecting life and regulating abortion, let’s see if it is putting a substantial obstacle in the path of a woman seeking an abortion.

Analytical Note on the Undue Burden Test

When the Supreme Court considers cases involving individual liberties, four issues exist:

Is there a fundamental right;

is the right infringed;

is the infringement justified by a sufficient purpose;

are the means sufficiently related to the end sought.

The undue burden test combines the latter three questions

Analytical Note on the Undue Burden Test continued

“undue burden” pertains to whether there is an infringement of the right, à but Note: the joint opinion in Casey also used it to analyze whether the law is justified.

Also Note: No level of scrutiny is articulated by the joint opinion in Casey;

GOVERNMENT REGULATIONS OF ABORTIONS

When may the government regulate abortions?

After Planned Parenthood v. Casey, the government can regulate abortions performed prior to viability

So long as there is not an undue burden on access to abortions.

Shouldn’t the father have a right too?

What specific state regulations of abortion are allowed? The Supreme Court reviewed constitutionality of various restrictions including

laws restricting partial-birth abortions;

waiting periods,

informed consent requirements,

fetal viability tests,

reporting and recording requirements,

the conduct of medical care personnel.

denial of government funding for abortions,

spousal consent and notification requirements, and

parental notice and consent requirements.

Laws Prohibiting “Partial Birth” Abortions

Stenburg v. Carhart (2000) (p. 879 text)

§516.1 The Court invalidated a state law banning an abortion procedure known as dilation and extraction (commonly referred to as “partial birth abortion”).

The Court in a 5-4 decision written by Justice Breyer, struck down a Nebraska statute which prohibited the performance of an abortion by a method known as dilation and extraction. Reaffirming the principles of Casey, the Court invalidated the law on two separate grounds:

1. The law did not have any exception for the life or health of the mother as required by Roe and Casey; and

2. The Court said the statute was worded so broadly that even though it was primarily aimed at preventing the dilation and extraction method of abortion, it could be applied to another more commonly used method, called dilation and evacuation. That application would impose an undue burden on a woman’s decision to terminate her pregnancy, thus running afoul of Casey.

Carhart II: Gonzales v. Carhart (Supplement)

Wikipedia.org:

Gonzales v. Carhart, 550 U.S. ___ (2007), is a United States Supreme Court case which upheld the Partial-Birth Abortion Ban Act of 2003. The case reached the high court after U.S. Attorney General Alberto Gonzales appealed a ruling of the United States Court of Appeals for the Eighth Circuit in favor of LeRoy Carhart that struck down the Partial-Birth Abortion Ban Act. Also before the Supreme Court was the consolidated appeal of Gonzales v. Planned Parenthood from the United States Court of Appeals for the Ninth Circuit, which had struck down the Partial-Birth Abortion Ban Act.

The Supreme Court's decision, handed down on April 18, 2007, upheld the federal ban and held that it did not impose an undue burden on the due process right of women to obtain an abortion, "under precedents we here assume to be controlling," such as the Court's prior decisions in Roe v. Wade and Planned Parenthood v. Casey. This case distinguished but did not reverse Stenberg v. Carhart (2000), in which the Court dealt with similar issues.

Decision

Justice Anthony Kennedy wrote for the Court that the respondents had failed to show that Congress lacked power to ban this abortion procedure. Chief Justice John Roberts along with Justices Samuel Alito, Clarence Thomas, and Antonin Scalia agreed with the Court's judgment, and they also joined Kennedy's opinion.

The Court left the door open for as-applied challenges, citing its recent precedent in Ayotte v. Planned Parenthood of New England. According to Washington Post reporter Benjamin Wittes, "The Court majority, following the path it sketched out last year in the New Hampshire case, decided to let the law stand as a facial matter and let the parties fight later about what, if any, applications need to be blocked."

The Court decided to "assume ... for the purposes of this opinion" the principles of Roe v. Wade and Planned Parenthood v. Casey. The Court then proceeded to apply those "principles accepted as controlling here."

The Court said that the lower courts had repudiated a central premise of Casey — that the state has an interest in preserving fetal life — and the Court held that the ban was narrowly tailored to address this interest. Relying deferentially on Congress's findings that this intact dilation and extraction procedure is never needed to protect the health of a pregnant woman, Kennedy wrote that a health exception was therefore unnecessary. And, where medical testimony disputed Congress's findings, Congress is still entitled to regulate in an area where the medical community has not reached a "consensus" — a finding at odds with the lower courts in this case.

The majority opinion held that "ethical and moral" considerations, including an interest in fetal life, represented "substantial" state interests which (assuming they do not impose an "undue" burden) could be a basis for legislation at all times during pregnancy, not just after viability. Thus, the Court believed that the pre-viability/post-viability distinction was not implicated in Carhart.

In addition, the Court distinguished the Stenberg case, which previously struck down Nebraska's partial-birth abortion law. The Court held that the state statute at issue in Stenberg was more ambiguous than the later federal statute at issue in Carhart.[2]

The majority opinion in Gonzales v. Carhart did not discuss the constitutional rationale of the Court's prior abortion cases (i.e. "due process"). However, the majority opinion disagreed with the Eighth Circuit that the federal statute conflicted with "the Due Process Clause of the Fifth Amendment [which] is textually identical to the Due Process Clause of the Fourteenth Amendment."

Concurrence

Justice Thomas filed a concurring opinion, joined by Justice Scalia, saving for another day the issue of whether Congress had sufficient power under the Commerce Clause to enact this ban. The Commerce Clause was also mentioned in the opinion of the Court, and was the only clause of the Constitution mentioned explicitly by the opinions in this case.

The concurrence also stated that Justices Thomas and Scalia joined the Court's opinion "because it accurately applies current jurisprudence." And, the concurrence reiterated their view that that current abortion jurisprudence "has no basis in the Constitution."

Dissent

Justice Ruth Bader Ginsburg dissented, joined by justices David Souter, John Paul Stevens, and Stephen Breyer, contending that the ruling was an "alarming" one that ignored Supreme Court abortion precedent. Justice Ginsburg's dissent was the only opinion in this case that mentioned the word "privacy". Justice Ginsburg, referring in particular to Planned Parenthood v. Casey, sought to ground the Court's abortion jurisprudence based on concepts of personal autonomy and equal citizenship rather than the Court's previous privacy approach: "Thus, legal challenges to undue restrictions on abortion procedures do not seek to vindicate some generalized notion of privacy; rather, they center on a woman's autonomy to determine her life's course, and thus to enjoy equal citizenship stature."

Justice Kennedy's opinion in Carhart did not touch upon the question of whether the Court's prior decisions in Roe v. Wade and Planned Parenthood v. Casey were valid. Dissenting Justice Ginsburg characterized this aspect of the Court's opinion as follows: "Casey's principles, confirming the continuing vitality of ‘the essential holding of Roe,’ are merely ‘assume[d]’ for the moment ... rather than ‘retained’ or ‘reaffirmed.’"

The difference is in Carhart I, if you pull a leg out and killed the fetus that way, it was covered. Under Carhart II, if the head came out first, then it would not be covered.

http://www.virginialawreview.org/inbrief/2007/07/23/weinsteintull.pdf

Waiting Periods

Planned Parenthood v. Casey

“Informed Consent” Requirements

Again, prior to Casey, the Supreme Court consistently invalidated informed consent requirements –

But after Casey such regulations are upheld.

Planned Parenthood v. Casey

Fetal Viability Tests (Not covered in text)

Webster v. Reproductive Health Services (1989) §513, p 172. The Court has held that states may prohibit the use of public facilities or employees to perform abortions unless necessary to save the life of the mother. The Court also upheld a state law that required testing and evaluation of fetal viability for all abortions performed after the twentieth week of pregnancy as that law was designed to protect state’s interest in potential human life because of a possible four-week margin of error in estimating gestational age.

Wagner: What if the state requires a fetal viability test? Under Casey, viability is still very important. Would that survive an undue burden challenge? Probably.

Reporting and Recording Requirements – undue burden? (Not covered in text)

Planned Parenthood of Central Missouri v. Danforth. (1976): This was a constitutionality challenge by Planned Parenthood to a Missouri law encompassing parental consent, spousal consent, clinic bookkeeping and allowed abortion methods. Portions of the challenged law were held to be constitutional, others not.

The Court generally upholds laws that require the recording and reporting of information concerning abortions so long as the information is protected as confidential.

Wagner: What about a waiting period? The old Supreme Court said that a waiting period was unconstitutional as there was no valid purpose. The new Supreme Court is recognizing an interest in the state for protecting pre-viability life.

Medical Procedures (Not covered in text)

Many states attempted regulated how doctors actually perform abortions. Almost all of these laws were declared unconstitutional.

However, these decisions were prior to Casey, and it is possible that under the undue burden test, the Court will be more inclined to allow such laws.

Government Restrictions on Funds and Facilities for Abortions

Maher v. Roe (1977) (p. 891 text) and Harris v. Mc Rae (1980) (p. 893 text). The Court held that neither a state nor the federal government has a duty to pay for abortions for indigent women, even if the government pays the expenses of childbirth for indigent women. The Court said that the government, through its refusal to subsidize welfare abortions, was placing no obstacle in the path of an indigent woman who wanted an abortion – her poverty was the obstacle. These cases are not true substantive due process cases because there was no government action mandatorily prohibiting abortions by indigent women. In other words, she still had a choice to exercise her right to terminate her pregnancy. The plaintiffs in these cases argued that non-funding of welfare abortions effectively prohibited women from exercising their right to terminate their pregnancies.

Poekler v. Doe (1977) The Court found that it was constitutional for a city to refuse to pay for non-therapeutic first trimester abortions in its public hospital.

Parental Notice Requirement for Unmarried Minors’ Abortions

Bellotti v. Baird (1979) (p. 901 text) The Court invalidated a Massachusetts law which required the consent of both parents before a minor could have an abortion. A plurality set forth rules for a judicial bypass to allow a minor to get an abortion without her parents’ notice or consent. If a state requires parental consent (one or both), it must also provide a confidential alternative procedure (usually a court hearing, but not necessarily) at which a judge makes one of two decisions:

1. If the minor is competent to make the abortion decision, she decides.

2. If she is not competent, due to age or mental status, the judge decides what her best interest is.

This is the parents’ right for the upbringing of their kids vs. The reproductive autonomy over a minor’s body.

Wagner: The underlying battle is does the Constitution create rights that the Court can manifest and say these are rights and they demand the government pay a benefit or is the view of the Constitution that the Bill of Rights provide a limit on government action?

CONSTITUTIONAL PROTECTION FOR MEDICAL CARE DECISIONS

The Right to Refuse Treatment A majority of the Court has recognized a low-level liberty interest in refusing medical treatment; so far, this has not been deemed to be a fundamental right.

Cruzan v. Director, Missouri Department of Health (1990) (p. 960 text) The Court upheld a Missouri statute that required proof by clear and convincing evidence that an incompetent person would have wanted to discontinue lifesaving food and water from being artificially administered. Nancy Cruzan had been in a persistent vegetative state for seven years when her parents sued to have the tubes removed. A majority of the Court found that there was a liberty interest in refusing unwanted medical treatment, but only four Justices deemed the interest fundamental.

Wagner: When it comes to contagious diseases, the government may have a compelling interest to overcome the right to refuse treatment to protect life. If the government is going to infringe on this interest they have to engage in procedural due process which means at the very least that there is clear and convincing evidence.

If you are unconscious, before the state takes your life they will need clear and convincing evidence that you really did want to exercise that right. The Court doesn’t really say what level of scrutiny should be used

No Right to Physician-Assisted Suicide The Due Process Clause of the Fourteenth Amendment does not create or protect a fundamental liberty interest in assisted suicide.

Washington v. Glucksberg (1997) ( p. 913 text) The Court unanimously upheld a State of Washington law which criminalizes assisting a person to commit suicide. The plaintiffs (including three seriously ill patients and four doctors) asserted a Due Process Clause liberty interest of a mentally competent, terminally ill adult to commit physician-assisted suicide. Relying heavily on history and legal tradition (mainly Anglo-American common law), the Court concluded that there exists no fundamental liberty interest in committing physician-assisted suicide. As part of its substantive due process analysis, the Court required a “careful description” of the asserted liberty interest. Rejecting broader characterizations of the asserted liberty interest (i.e., the “right to die,” or the “liberty to shape death”) the Court concluded that the proper tradition to consult was that concerning physician-assisted suicide. After concluding that no such fundamental liberty interest existed, the Court found that Washington’s asserted state interests, including preserving human life and avoiding a possible slide toward voluntary and involuntary euthanasia, easily met the rational-basis test of being legitimate government interests.

Wagner: The interests that the government has are: preserving human life, the slippery slope of euthanasia, the Hippocratic oath, etc.

The other side’s argument is: This is a fundamental right (so you can make the Court use strict scrutiny) and will be struck down. They tried to hook it onto the Fourteenth Amendment liberty interest. The Court rejected the argument – no fundamental right; therefore, you apply the rational relation test and the statute survives.

If you are Scalia and you are looking for a fundamental right in the Constitution, you look at history and tradition of the founders. They found no fundamental right and the legislation will survive. This has progressed in the Netherlands to the extent that the doctors weren’t even getting consent from the patients or even the parents for assisted suicide.

No Equal Protection Violation When a State Allows a Patient to Refuse Lifesaving Medical Treatment But Prohibits Physician-Assisted Suicide. Under the Equal Protection Clause, there is a rational distinction between a patient refusing unwanted lifesaving medical treatment and a patient receiving assistance in committing suicide.

Vacco v. Quill (1997) (p. 906 text) The Court unanimously ruled that New York does not violate the Equal Protection Clause by allowing competent adults to refuse unwanted lifesaving medical treatment, but prohibiting assisted suicide. The Court said that since a prohibition on assisted suicide does not abridge any fundamental right (See Glicksberg), or discriminate against any suspect or quasi-suspect class, rational basis scrutiny is called for under equal protection analysis. Disagreeing with the Second Circuit Court of Appeals, the Court saw a clear distinction between refusing unwanted medical treatment (thus letting nature take its course), and taking affirmative steps to bring about death (by allowing a doctor to prescribe life-ending medication). The Court recognized a number of state interests that justified its ban on assisted suicide, among them preserving life, avoiding a possible slide towards euthanasia, and maintaining a physician’s role as a healer.

Bar Review Hypo: in 1994 Oregon became the first state to legalize assisted suicide when it enacted the Oregon Death With Dignity Act (ODWDA), exempting from criminal or civil liability state-licensed physicians who, in compliance with specific safeguards, dispense or prescribe a lethal dose of drugs upon the request of a terminally-ill patient.

In response, Congress says there is no fundamental right. If the statute survives under strict scrutiny, the federal statute (Controlled Substances Act) is inconsistent. If the federal law preempts the state law, what does an Oregon doctor do – prescribe a gun and a bullet? Amend drug laws under Commerce Clause? Anyone who uses drugs that crossed state lines will be guilty.

Wagner: When the court says we have no fundamental right to assisted suicide, it’s important to know how we got there. If the government has found a power source to pass a law that says, assisting someone to commit suicide is against the law. If you are a physician then you have to go to court and you are going to acknowledge that the government has a legitimate power source here whatever it is. The second question you always ask is, does some other part of the Constitution limit that right? That’s when you go through the amendments looking for that right. If there is no fundamental right it will survive

CONSTITUTIONAL PROTECTION OF SEXUAL ACTIVITY

State prohibition of adult consensual homosexuality activity violates the Due Process Clause. The Court held that a state statute making it a crime for two persons of the same sex to engage in certain intimate sexual conduct is an abridgement of liberty protected under the DPC.

Lawrence v. Texas (2003) (p. 920 text) The Court with five justices relying on a substantive due process rationale, invalidated the application of a Texas anti-sodomy statute to adult males who had engaged in a consensual sexual act in the privacy of a home. In reaching its result, the Court overruled Bowers v. Hardwick (1986), in which the Court upheld the application of a Georgia criminal sodomy statute to consenting adults who engaged in homosexual activity. In Bowers, the Court rejected the argument that the constitutional right of privacy was broad enough to protect this activity, and also rejected the claim that this activity was protected because it took place in the home. In Lawrence, the majority said that the liberty protected by the Constitution allows homosexuals the right to choose to enter relationships in the confines of their homes and their private lives and retain their dignity as free persons. The majority does not say that there is a fundamental right to engage in homosexual activity, nor does it apply the traditional strict scrutiny standard. Rather, it concludes that Texas has no legitimate state interest underlying its law (rejecting the morality argument), and thus the Texas statute fails rational basis scrutiny. Justice O’Connor agrees that the Texas statute is unconstitutional, but relies on the Equal Protection Clause to reach her conclusion. Refusing to join the majority’s overruling of Bowers, she instead says that a law branding one class of persons as criminal based solely on a state’s moral disapproval of that class violates the Equal Protection Clause.

Wagner: The Justices have a different vision as their role as Justices (different pair of sunglasses example). Ask yourself, what do I believe and why do I believe it? Don’t be afraid to ask, what if I am wrong? If you can do that, you can look very carefully at both sides taking out the politics. If you can’t agree on the question, you can’t compromise on it.

How you address the issue will determine whether or not the Supreme Court will hear the case. Example: Whether or not life has value at all stages that the government has a right to protect or whether or not a person has a right to end their life if they want to.

For Exam: Explain Why. Wagner needs a stamp for exams that reads: “Explain Why.”

Other Constitutional Protections

Constitutional protection for the right to travel

Voting and rights of political participation

Constitutional protection for access to the courts

Constitutional protection of a right to education

Constitutional Law II – Week 4

EQUAL PROTECTION

Equal Protection of the Laws:

1] Introduction to EP Analysis

2] Non-suspect Classifications

3] Classifications Based on Race and National Origin

“No state shall deny to any person within its jurisdiction the equal protection of the laws.”

Wagner: First ask yourself, does the law classify a person explicitly or implicitly? If the law Congress is passing is classifying people, then you apply an equal protection analysis.

Example: All people with blonde hair have to take the final exam.

If the law Congress is passing doesn’t classify people (applies to all people), then you apply substantive due process analysis to see whether it violates the Constitution.

Equal protection issues can be broken down into three questions:

1. What is the classification?

2. What level of scrutiny should be applied?

3. Does the particular government action meet the level of scrutiny?

For Analysis: We start out by seeing how the law affects people.

Question 1: What is the Classification?

Equal protection analysis always must begin by identifying how the government is distinguishing among people.

Two ways of establishing a classification:

1. Where the classification exists on the face of the law (i.e., where the law in its very terms draws a distinction among people based on a particular characteristic).

2. Where the law is facially neutral, but there is a discriminatory impact; or there is a discriminatory effect from its administration.

Discriminatory impact is insufficient to prove a racial or gender classification.

If law is facially neutral, demonstrating a race or gender classification requires proof of a discriminatory purpose behind the law.

Example: Must be 16 years of age to get a drivers license – a classification based on age. Is it constitutional? To attack it, use equal protection.

Suspect classification: Classification of individuals based upon race or gender – things you cannot change. The classification becomes “suspicious.”

There is no reason why age, wealth (for welfare benefits), bar passage, etc. would be as suspect.

Question 2: What is the appropriate Level of Scrutiny based upon the classification that the law creates?

The level of scrutiny applied by the Court depends on the type of discrimination

Strict Scrutiny: Discrimination based on race or national origin is subjected to strict scrutiny­ – a very, very important reason.

Under strict scrutiny a law is upheld if it is proved necessary to achieve a compelling government purpose.

- The government must have a truly significant reason for discriminating, and

- It must show that it cannot achieve its objective through any less discriminatory alternative.

The government has the burden of proof under strict scrutiny

Intermediate Scrutiny: Intermediate scrutiny is used for discrimination based on gender and for discrimination against non-marital children (kids cannot control whether his or her parents are married) – an important reason.

Under intermediate scrutiny a law is upheld if it is substantially related to an important government purpose.

Under intermediate scrutiny, the government has the burden of proof.

Rational-Basis Scrutiny: Where there is no special reason why the Court should be suspicious of the classification - the law receives a rational-basis level of scrutiny

Under rational basis review, a law will be upheld if it is rationally related to a legitimate government purpose.

The challenger has the burden of proof under rational basis review.

Sometimes you look at a law and on its face it appears to be neutral and it looks like it treats everybody the same, but at closer look, it has a discriminatory impact or discriminatory effect in its administration.

For example: Say a police department passes a regulation that says all police officers have to be 5’-10” and 170 pounds. Facially neutral, applies to everybody. A lot more men are going to qualify than women are.

The discriminatory impact is not enough to create a classification. You have to have discriminatory impact and you need to show discriminatory intent or purpose behind the law.

Criteria applied by Court in determining appropriate scrutiny level:

  • Whether characteristics are immutable. Is this something the person can control? Can they control that they were born a certain race? The country they were born in? Born male or female? If it is immutable, the Court will apply strict scrutiny. This is based upon the individual.

  • Whether the group has the ability to protect itself via the political process. Does a person born in another country have the ability to participate in the political process? The more they are able to, the less likely that the Court will raise the level of scrutiny. This is based upon a group.

  • Whether a history of discrimination against the group exists. Example: there is a history that blacks were discriminated against in this country. The Court will elevate the level of scrutiny. This is another group characteristic.

Wagner: The government can discriminate based on race if it has a compelling reason (i.e. national security) but the Court will apply strict scrutiny and most of the time the law is struck down. Under intermediate scrutiny the law is only going to be upheld if the government can show that it is substantially related to an important government purpose.

Most of the time when the government passes a law, there will not be a reason for the Court to be as suspicious of the classification and get rational-basis review – is the classification related to a legitimate government purpose.

Question 3: Does the Government Action Meet the Level of Scrutiny?

The level of scrutiny is the rule of law that is applied to the particular government action being challenged as denying equal protection.

In evaluating the constitutionality of a law, the Court evaluates both the law’s ends and its means.

- For strict scrutiny the end must be deemed compelling for the law to be upheld;

- For intermediate scrutiny the end has to be regarded as important; and

- For the rational basis test there just has to be a legitimate purpose.

Spectrum: Can the law be over-inclusive or under-inclusive? A law is under-inclusive if it doesn’t apply to everybody that is similarly situated – it doesn’t apply to everyone it could to fight the problem.

Example: Are there some 15 year olds who are mature enough to drive? Are there some 17 year olds who ought not to have drivers’ licenses?

When the Court looks at that statute on whether to uphold the equal protection clause, there is a lot of over-inclusiveness and under-inclusiveness but the Court is going to allow it because this is just a classification that isn’t treated as suspect (age).

On the other hand, if the classification were based on race, the Court would not tolerate that amount of under or over-inclusiveness.

Rational-basis Intermediate Strict


Wagner: Refer to charts on TWEN page.

The Protection of Fundamental Rights Under Equal Protection

Use fundamental rights strand of EP analysis if legislature establishes a classification system discriminating against a certain group of persons depriving them of a fundamental right

Note: it is the deprivation of the fundamental right that increases the scrutiny level - NOT the classification

Example: Sterilization hypo of three-time looser chicken thief – snip, snip. The fundamental right was the right to procreate. That statute also classifies people who have committed three crimes and people who have not committed three crimes. It’s not based on race or gender so it would normally get rational-basis scrutiny but, it is discriminating against people as to their fundamental right to procreate; therefore, a strict scrutiny level of review.

Review

Three tiers of equal protection scrutiny

Strict scrutiny is applied to a law that discriminates based on race, national origin, alien status, and to any classification that substantially impairs exercise of a fundamental right

Intermediate scrutiny is applied to a law that discriminates based on gender or illegitimacy

Rational basis scrutiny is applied to a law that classifies on any other basis

PROFESSOR WAGNER’S ANALYTICAL STRATEGY

Equal protection analysis is essentially grafted over due process analysis

Always start analysis by asking whether law classifies persons, explicitly or implicitly

If answer is yes, then do equal protection analysis

If answer is no, then go to due process analysis

Equal protection analysis will send you to due process analysis if classification is not suspect or quasi-suspect but infringes the exercise of a fundamental right

Part 1: Does the law classify persons?

If yes, go to Part 2

If no, go to due process analysis

Part 2: Is the purpose of the law to discriminate against the class and does it have a discriminatory impact upon the class?

If yes to both, go to Part 3

If no to one or both, analysis is rational basis

Part 3: Is the classification based on race, national origin, or alien status?

If yes (apply strict scrutiny), then:

3a: Does the government have a compelling interest to justify the classification?

If no, then classification violates equal protection

If yes, then go to Part 3b

3b: Is the classification necessary or narrowly tailored to meet the government’s compelling interest?

If no, then classification violates equal protection

If yes, then no equal protection violation

If no, then go to Part 4

Part 4: Is the classification based on gender or illegitimacy?

If yes (apply intermediate scrutiny), then:

4a: Does the government have an important interest to justify the classification?

If no, then classification violates equal protection

If yes, then go to Part 4b

4b: Is the classification substantially related to the government’s important interest?

If no, then classification violates equal protection

If yes, then no equal protection violation

If no, then go to Part 5

Part 5: Does a non-suspect classification impair the exercise of a fundamental right?

Use modified due process model to answer

5a: Is it a fundamental right?

If no, classification is judged using rational basis test

If yes, go to Part 5b

5b: Does the classification substantially impair the exercise of the right?

If no, classification is judged using rational basis test

If yes, go to Part 5c

5c: Does the government have a compelling interest to justify the classification?

If no, then equal protection violation

If yes, then go to Part 5d

5d: Is the classification necessary or narrowly tailored to meet that compelling interest?

If no, then equal protection violation

If yes, then no equal protection violation

Discrimination Against An Individual Is Sufficient for a Claim

Equal protection does not require allegations that the government has discriminated against a group or on the basis of group characteristics

There is a claim under the equal protection clause even for discrimination against a “class of one.”

Example: You can be the only guy who has to give an easement of 30 feet to the city when everyone else has to give only 15 feet. It is government action and it’s a classification. You may not have much of a case under rational basis, but you’re not going to be thrown out of court for failure to state a claim.

Equal Protection and Formulations of the Rational Basis Test

The rational basis test is the minimal level of scrutiny that all government actions challenged under equal protection must meet.

In other words, unless the government action is a type of discrimination that warrants the application of intermediate or strict scrutiny, rational basis review is used.

Although the Court has phrased the test in different ways, the basic requirement is that a discriminatory law meets rational basis review if it is rationally related to a legitimate government purpose.

The equal protection clause is satisfied so long as the classification is “rationally related to a legitimate state interest.”

THE RATIONAL BASIS TEST


The Requirement for a “Legitimate Purpose”

What Is a “Legitimate” Purpose?

Protecting safety, public health, or public morals

Virtually any goal that is not forbidden by the Constitution will be deemed sufficient to meet the rational bias test.

Must it be the actual Purpose or is Any Conceivable Purpose Enough?

The Court is willing to accept any conceivable legitimate purpose as sufficient, even if it is not the government’s actual purpose.

U.S. Railroad Retirement Board v. Fritz (1980) (630 text) The Court upheld under rational-basis review, a federal retirement law that prevented certain employees from receiving “double-dip” retirement benefits. Under the law, a person qualified for the windfall benefits only if he or she was active in the railroad business as of 1974. As a result, a person with 24 years of experience would not qualify for the windfall unless he or she worked for the railroad in 1974, whereas a person with 11 years of experience would qualify for the windfall of he or she worked with the railroad in 1974. The workers with more experience challenged the law under the equal protection component of the DPC of the Fifth Amendment. The Court found that Congress had not acted arbitrarily or irrationally, and said that it would uphold a law if there were any plausible reason for it, even though Congress may not have relied on the reason for passing the law.

The Requirement for a “Reasonable Relationship”

The Court Gives Deference to Government in Determining If There Is a Reasonable Relationship

“most relaxed and tolerant form of judicial scrutiny”

Ask whether the classifications drawn in a statute is “reasonable in light of its purpose”

Under the rational basis test, laws upheld unless government’s action is “clearly wrong, a display of arbitrary power, not an exercise of judgment.”

Tolerance for Under-inclusiveness / Over-inclusiveness

Tolerance for Under-inclusiveness

Railway Express Agency, Inc v New York (1949) (634 text) The Court upheld, under rational basis review, a New York City traffic regulation which prohibited advertising for hire on vehicles on New York streets, but allowed the owner of a vehicle to advertise his or her own business on the vehicle. The Court accepted the city’s argument that it was trying to enhance the safety of the streets, and set forth the rule that, at rationality review, a government may deal with a problem a bit at a time. The government may have concluded that truck owners who advertise their own goods pose less of a safety problem than those who advertise for hire. This case came down well before the Court gave First Amendment protection to commercial speech.

Tolerance for Over-inclusiveness

New York Transit Authority v. Beazer (1979) (p. 637 text) The Court upheld under rational basis review, a Transit Authority rule which excluded methadone users from any employment with the Authority. This ban included non-safety sensitive jobs, and was upheld in the face of the statistical fact that 75% of those who have been on methadone for at least one year are free from illicit drug use. At rationality review, a law does not have to be narrowly tailored (or even substantially related to achieving the law’s goal), and may be over-inclusive in that it disadvantages too many people.

Sometimes law so over-inclusive / under-inclusive that Court deems law Arbitrary and Unreasonable

U.S. Dept of Agriculture v. Moreno (1973) (640 text) Lexis: Appellees consisted of several groups of individuals who alleged that, although they satisfied the income eligibility requirements for federal food assistance, they were nevertheless excluded from the program solely because the persons in each group were not all related to each other. The district court held that the "unrelated person" provision of 7 U.S.C.S. § 2012(e) created an irrational classification in violation of the equal protection component of the Due Process Clause of the Fifth Amendment. On certiorari, the Court affirmed, holding that the "unrelated person" provision was irrelevant to the state purpose of the Food Stamp Act and did not operate to rationally further the prevention of fraud. The classification acted to exclude not only those who were likely to abuse the program, but also those who were in need of the aid but could not afford to alter their living arrangements so as to retain their eligibility. OUTCOME: The Court affirmed the district court's determination that the unrelated household classification was invalid because it was irrelevant to the stated purpose of the Food Stamp Act and was not rationally related to furthering any legitimate government interest.

Other Rare Example Where Court Deems Law Arbitrary and Unreasonable:

Laws that discriminate against the mentally retarded or the mentally ill receive rational basis review – the plaintiff has the burden of showing that there is no legitimate reason for the law, or that the means are not rationally related to the asserted government interest.

City of Cleburne v Cleburne Living Ctr. (1985) (643 text) The Court invalidated, under rational-basis review, a city’s denial of a special use permit for the operation of a group home for the mentally retarded. The Court refused to afford the mentally retarded the status of a quasi-suspect or suspect class. The Court said that the mentally retarded are a very diverse class of people, and the lack of homogeneity within the class (mildly retarded to profoundly retarded) made this the kind of group of people best dealt with by a legislature. The Court was also unwilling to begin adding to the list of quasi-suspect or suspect classes. If the mentally retarded were given special protection by the Court, why not the mentally ill, the infirm, or the disabled? On the facts, the Court invalidated the law because its only real basis appeared to be antipathy toward the mentally retarded.

CLASSIFICATIONS BASED ON RACE

Race Discrimination & Slavery before the 13th and 14th Amendments

Constitutional provisions expressly protected slavery.

- Article I, §2, requires apportionment of the House based on the “whole number of
free Persons” and “three fifths of all other Persons.”

- Article I, §9, prevented Congress from banning importation of slaves until 1808,
and Article V prohibited this provision from being altered by constitutional
amendment.

- Article IV, §2, contains the fugitive slave clause

Court Decisions Protecting Slavery:

Prigg v. Pennsylvania

State v. Post

Dred Scott v. Sandford (1856) (p. 649 text) Slaves are property and therefore Dred Scott has no standing even though he is in a non-slave free state (Illinois). This was the first time since Marbury v. Madison that the Supreme Court has found a statute (Missouri Compromise) unconstitutional. This case was decided even though the Court did not have jurisdiction.

The Post-Civil War Amendments that limit government action:

Thirteenth Amendment:

- Prohibits slavery and involuntary servitude

Fourteenth Amendment:

- Overruled Dred Scott - declared all persons “born or naturalized in the United States…are citizens of the United States and of the State wherein they reside.”

- Guaranteed that no state shall...deprive any person of life, liberty, or property without due process of law, or deny any person “equal protection of the laws.”

Wagner: You can write laws, but you have to give them equal protection of the laws – strict scrutiny for racial classification.

Strict Scrutiny for Laws w/ Racial Classifications

Racial classifications allowed only if government can meet heavy burden of demonstrating discrimination is necessary to achieve a compelling government purpose.

- The government must show extremely important reason for its action;

- The government must demonstrate that goal cannot be achieved through any less discriminatory alternative.

All racial classifications (whether disadvantaging or helping minorities) must meet strict scrutiny.

Policy on strict scrutiny for race: (1) Look at the history and the way the government treated race in the past; (2) This is an immutable characteristic that the individual has no control over; and (3) Previously unable to vote/lack of political clout.

Proving the Existence of a Race Classification – how do you know you have a race-based classification?

First Way: If law expressly discriminates based on race (or national origin), show the classification exists on the face of the law

Second Way: If law is facially neutral, prove a race classification exists by demonstrating discriminatory administration or discriminatory impact. NOTE: this requires proof of a discriminatory purpose.

First Way – Face Of The Law:

Three Types of Classifications Based on Race on the Face of the Law:

1. Race–specific Classifications that Disadvantage Racial Minorities

2. Racial Classifications Burdening Both Whites and Minorities

3. Laws Requiring Separation of the Races

Plessy v. Ferguson (1896) (p. 663 text) “Separate But Equal” Constitutional (Train cars).

Brown v. Board of Education (1954) (p. 667 text) “Separate But Equal” Unconstitutional. The Court ruled that separate educational facilities for children in public schools are inherently unequal. The Court said that such segregation violates the Equal Protection Clause even if the physical facilities and other tangible benefits may be equal. While public school education is not a fundamental right, it is preservative of other basic interests such as the right to participate in the processes of government. Enforced separation of the races not only reinforces a sense of inferiority on the part of black school children, but also severely affects their motivation to learn.

Wagner: Separate but equal can never be equal. What matters most before the Supreme Court is what the question is. If you can frame the issue, you win. If the Court had simply said that the classrooms are separate but equal, Brown would have been just another case. The issue became, whether segregation is constitutional or not and the Court said no. That doesn’t happen unless the Court asks the question that way.

Two strategies to overcome this:

1. If the rule is separate but equal, we will make the school equal and cost the state $$$.

2. Separate but equal can never be equal.

· Johnson v. California (2005) (Supplement) Lexis: PROCEDURAL POSTURE: Petitioner prison inmate sued respondent California Department of Corrections (CDC), alleging that the CDC's unwritten policy of segregating new and transferred prisoners by race violated the inmate's constitutional right to equal protection of the laws. Upon the grant of a writ of certiorari, the inmate appealed the judgment of the U.S. Court of Appeals for the Ninth Circuit which upheld the policy under a deferential standard. OVERVIEW: Under the CDC policy, a new or transferred prisoner at a correctional facility was assigned a cellmate of the same race in a reception center pending a regular housing assignment. The CDC contended that the policy was necessary to prevent violence caused by racial prison gangs and was thus reasonably related to legitimate penological interests. The United States Supreme Court held, however, that the policy was instead subject to strict judicial scrutiny since it was based on racial classification, and thus the classification was required to be narrowly tailored to further compelling CDC interests. All racial classifications were immediately suspect and subject to strict scrutiny, and the CDC policy was no exception simply because all races were equally segregated, since the segregation itself reinforced racial and ethnic divisions in the correctional facilities. Further, the CDC's discretion and expertise in the unique area of managing daily prison operations did not warrant deference to the CDC's use of race as a means of controlling prison violence, and compromising the inmate's equal protection rights was not necessarily needed for proper prison administration. OUTCOME: The judgment upholding the CDC's racial segregation policy under a deferential standard was reversed, and the case was remanded for a determination of the constitutionality of the policy under the standard of strict scrutiny.

Second Way – Demonstrate Discriminatory Administration or Discriminatory Impact:

If a law is facially neutral with regard to race, then proving racial classification requires demonstrating both:

Discriminatory impact

  • Palmer v. Thompson (1971) (p. 684 text) The Court affirmed that the city's decision to close all swimming pools did not violate the Thirteenth or Fourteenth Amendments. Substantial evidence supported the city's claim that the pools were closed to maintain peace and order and because the pools could not be operated economically on an integrated basis. Moreover, there was no indication that the city was involved in maintaining private, segregated pools.

And discriminatory purpose

  • Washington v. Davis (1976) (671 text) The Court upheld the validity of a test used to screen applicants for jobs as police officers in the District of Columbia, even though the test had a racially disproportionate impact. The statistical evidence alone was not enough to make out a prima facie case of an unequal protection argument; intent was also needed and could not be inferred from language of the test itself, or from its application.

  • McCleskey v. Kemp (1987) (674 text) The Court rejected the Eight Amendment argument of a defendant sentenced to death who offered a study showing that defendants who murder white victims are 4.3 times more likely to receive the death penalty as those charged with killing black victims. Given the protections built into the Georgia capital sentencing system, the Court held that the study was not sufficient to prove a constitutionally significant risk of racial bias in capital sentencing cases.

Wagner: In to show discriminatory intent under strict scrutiny, you have to show that when they wrote the statute they were intending on discriminating someone. Juries: You have to show that the jury intended to discriminate when they sentenced someone. Prosecutor: Have to show that when the defendant was indicted the prosecutor intended to discriminate on based on race. It’s tough to prove. If you cannot make your case, you haven’t made a constitutional case. You may still have a remedy under a federal or state discrimination statute.

  • Personnel Administrator of Mass v. Feeney (1939) (686 text) The Court upheld a Massachusetts law giving a lifetime preference on civil service exams to veterans, even though 98% of the beneficiaries of the law were men. While rejecting the gender based equal protection challenge, the Court said that the degree of disparate impact, as well as its foreseeability are relevant factors in determining intent.

  • Arlington Heights v. Metro Housing Development (1977) (688 text) The Court rejected an equal protection challenge to a village zoning ordinance which had the effect of prohibiting racially integrated low-income housing. No racially discriminatory intent was shown. The Court said that a plaintiff need not show that racial animus was a motivating factor behind the government action. There are a number of things to consider when trying to prove intent: the statistical impact of the law on a particular group; a pattern or effect of the law unexplainable on grounds other than race; the legislative or actual history of the government action; or a departure from normal procedural sequence. While none of these alone may be dispositive of intent, a court will consider the totality of circumstances in deciding the intent question.

Note: If plaintiff produces evidence of a discriminatory purpose, the burden shifts to the government to prove it would have taken the same action without the discriminatory motivation.

Wagner: If the Court strikes something down because of discriminatory intent, Congress will find some straw purpose that will suffice the next time.

ANALYTICAL STRATEGY

Analytical Strategy for Second Way Discriminatory Impact and Discriminatory Purpose:

If the law is racially neutral, challenger must show a discriminatory purpose and a discriminatory effect

If such proof is provided, the government has the opportunity to demonstrate it would have taken the same action regardless of race or national origin.

If the Court accepts the government’s justification and rejects the claim of a discriminatory purpose, only rational basis review is used.

If the Court is convinced that a discriminatory purpose exists, the Court treats the law as a race or national origin classification and the law will be invalidated.

Formal application of strict scrutiny unnecessary since persuading the Court that the purpose behind the law is discriminatory forecloses the government’s ability to show a compelling purpose for it.

Wagner: When you see a classification (after proving it’s a classification), the question becomes, what level of scrutiny does the Court give it? How close is the Court going to look at that law when it decides whether or not it violates the Equal Protection Clause?

RACIAL CLASSIFICATIONS BENEFITING MINORITIES


Preferential Racial Relief /Affirmative Action

What level of scrutiny should be used for racial classifications benefiting minorities?

What purposes for affirmative action programs are sufficient to meet the level of scrutiny?

What techniques of affirmative action are sufficient to meet the level of scrutiny?

What level of scrutiny should be used for racial classifications benefiting minorities?

Strict scrutiny used to evaluate all government affirmative action plans:

Richmond v. J.A. Croson Co. (1989) (708 text). When reviewing a state-based affirmative action program, courts will apply strict scrutiny. In addition, states may not use societal discrimination as a basis for enacting an affirmative action program.

Adarand Constructors v. Pena (1995) (716 text). After the Adarand decision, federal race-based affirmative action programs receive strict scrutiny – the same standard the Court uses when reviewing state race-based affirmative action programs.

Grutter v. Bollinger (2003) (p. 722 text) The Court, by a 5-4 vote, upheld the admissions policy of the University of Michigan Law School which considered race or ethnicity as a ”plus” factor in an applicant’s file, rather than making it the decisive factor in the admissions decision. The Court, applying strict scrutiny, held that diversity in the classroom is a compelling government interest, and that the Law School’s use of race was narrowly tailored to achieving that goal. The Court accepted the Law School’s argument that it needed a “critical mass” of minority students to assure that the diversity goals of cross-racial understanding and the breaking down of racial stereotypes would be met. Justice O’Connor, for the majority, focused on the highly individualized, holistic review of an applicant’s file, distinguishing this approach from mechanistic use of a point system in Gratz.

Gratz v. Bollinger (2003) (P. 740 text). The Court ruled that a university may assert a compelling interest in assembling a racially and ethnically diverse student body, but that a point system which operated to guarantee admission to minority applicants was not narrowly tailored to achieve that goal. The main reason why the point system was found to fail the narrowly-tailored requirement was that the 20 point distribution to each member of an underrepresented minority group made the factor of race decisive (not just

a “plus”) for virtually every minimally qualified underrepresented minority applicant.

What purposes for affirmative action programs are sufficient to meet the level of scrutiny?

Remedying Specific Past Discrimination – Yes (specific) No (generic)

Enhancing Diversity – Yes

Providing Role Models – No

Enhancing Services Provided to Minority Communities – No

What techniques of affirmative action are sufficient to meet the level of scrutiny?

Set-asidesAllowed, if at all, only to remedy clearly proven past discrimination

Using Race as One Factor in Decisions to Help Minorities – Yes

Deviations From Seniority Systems – No

Drawing Election Districts to Increase Representation –Yes if politics is motive and not race (Most blacks are democrats example).

CLASSIFICATIONS BASED ON SEX

Intermediate Scrutiny for Laws with Gender Classifications

Intermediate scrutiny is the appropriate level of review for gender classifications:

To withstand a constitutional challenge, classifications by gender must:

1. Serve important governmental objectives; and

2 Must be substantially related to those objectives.

Craig v. Boren (1976) (758 text). Drinking Age. The Court, adopting the intermediate level of scrutiny, invalidated a law which discriminated against males by requiring them to be 21 years old to buy beer, when females could buy it at age 18. The idea underlying this prospective law (prospective because it attempted to prevent future misconduct rather than punish past misdeeds) was that 18-20 year old males were likely to drink, drive, and cause accidents; however, only about 2% of that group actually engaged in the undesired behavior, making the law about 98% overinclusive (Wagner: both men and women have the same amount of accidents at that age) – too much for intermediate scrutiny. At intermediate scrutiny, the means chosen by the government must be substantially related to achieving the goal of the law. The Court will not tolerate a high degree of overinclusiveness in cases of gender discrimination.

United States v. Virginia (1996) (761 text). Military School. The Court ruled that the Equal Protection Clause was violated by the Virginia Military Institute, state military college, excluding women from attending the school. The Court said that VMI would have to demonstrate an “exceedingly persuasive justification” for its male only admissions policy, showing at least that the categorical exclusion of women from the student body was substantially related to an important governmental objective. The Court rejected Virginia’s proposed remedy of creating a separate, all-female military training school in the state.


Proving the Existence of a Gender Classification

First Way: If law expressly discriminates based on gender, show the classification exists on the face of the law

Craig v. Boren (1976) (758 text)

United States v. Virginia (1996) (761 text)

Second Way: if law is facially neutral, prove a gender classification exists by demonstrating discriminatory administration or discriminatory impact,

NOTE: this requires proof of a discriminatory purpose. You have to prove that they passed this statute with the intent to discriminate against women:

- Personnel Administrator of Massachusetts v. Feeney (1979). The Court upheld a Massachusetts law giving a lifetime preference on civil service exams to veterans, even though 98% of the beneficiaries of the law were men. While rejecting the gender-based equal protection challenge, the Court said that the degrees of disparate impact, as well as its foreseeability are relevant factors in determining intent.

Note: The ways of proving a discriminatory purpose based on gender are identical to the ways of proving a discriminatory purpose based on race.

CLASSIFICATIONS BASED ON ALIENAGE


Strict Scrutiny for Laws with Alienage Classifications

General Rule: Alienage classifications allowed only if government can meet heavy burden of demonstrating discrimination is necessary to achieve a compelling government purpose.

Government must show extremely important reason for its action;

Government must demonstrate that goal cannot be achieved through any less discriminatory alternative.

e.g., Graham v. Richardson (1971) (790 text) A state cannot deny Welfare Benefits to resident aliens. The Court invalidated on supremacy grounds, state statutes which denied welfare benefits to resident aliens. The Court said that such laws conflict with overriding national policies in an area (the regulation of immigration) constitutionally entrusted to Congress.

Wagner: An alien is a non-citizen. If you are an alien in this country lawfully, the court will pull out its microscope and look at it with strict scrutiny.


EXCEPTIONS to the General Rule of Strict Scrutiny

Alienage Classifications Related to Self-Government and the Democratic Process:

Foley v. Connelie (1978) (792 text) (rational basis test used) A state can exclude aliens from being police officers. The Court upheld, under rational basis review, New York’s exclusion of aliens from its police force. The Court reasoned that police perform discretionary functions which are essential to the carrying out of public policy and the laws of the state.

Wagner: if it’s within the constitutional prerogatives of the state (such as hiring police officers), and it is related to self government and the democratic process, the Court will apply rational basis. Make sure you know this. If an exception is going to show up, this is one of the areas where it does show up.

Federally (as opposed to state) Approved Discrimination (e.g. by Congress or Presidential order):

Mathews v. Diaz (1976) (rational basis test used) Congress may impose restrictions on an alien’s eligibility for welfare benefits. The Court upheld a federal law imposing restrictions on an alien’s ability for Medicare benefits. The Court said that because alienage issues touch on our country’s relations with foreign countries, the Court should be extremely differential to any decision of Congress or the President dealing with aliens. The Court likened the Court’s role in federal alienage cases to its role in political question cases.

Wagner: The Court defers to the government in the areas of immigration. A federal regulation creating a class will only get a rational basis test.

Illegal Undocumented Aliens, Equal Protection, and Public Education for Children:

Plyler v. Doe (1982) (799 text) The level of scrutiny not articulated but it appears that intermediate scrutiny will be used at least in evaluating government actions discriminating against undocumented immigrants regarding education.

Wagner: Children cannot control where or to whom they were born – it’s an immutable characteristic, so maybe it doesn’t get strict scrutiny but it might get a watered-down version of it in intermediate scrutiny.

OTHER TYPES OF DISCRIMINATION

Rational Basis Review

Examples:

Age Classifications

Discrimination based on Disability

Wealth Discrimination

Discrimination based on Sexual Orientation

One can challenge as denying equal protection:

Each is subjected only to rational basis review, unless the discrimination was with regard to race, national origin, gender, alienage, or legitimacy.

WEEK 6 – FIRST AMENDMENT

Wagner: “You can’t have a functional democracy without free speech.”

Constitutional Law II

Assignment 6: Freedom of Expression


Free Speech Methodology

Content Neutrality

Vagueness

Overbreadth

Prior Restraints

The First Amendment

“Congress shall make no law respecting an establishment of religion, or prohibiting the free exercise thereof; or abridging the freedom of speech, or of the press; or the right of the people peaceably to assemble, and to petition the Government for a redress of grievances.”

Wagner: The first question you ask is, where am I going to go to ask the Court to strike down the law? Depending on the case, it could be vague, or overbroad. It could be vague and overbroad. It could be a time, place, and manner restriction (which we haven’t covered yet).

Purposes of the First Amendment

First Amendment was clearly meant:

prohibit licensing of publication such as existed in England;

to forbid punishment for seditious libel; and

to make further prosecutions for criticism of the government (w/o any incitement to law-breaking), forever impossible in US.

Other than the plain language of the Amendment, little indication exists of what the framers intended beyond the above two purposes.

Why is Speech Protected?

Theories why freedom of speech should be regarded as a fundamental right:

to further self-governance and the democratic process;

to aid the discovery of truth via the marketplace of ideas;

to promote autonomy; and

to foster tolerance.

Wagner: The “why” is the justification of why it’s a fundamental right.

Free Speech Methodology

Main concept: As part of the analysis of a speech regulation, you must consider these initial questions:

First, does the regulation appear to be concerned with the content of the speech?

Second, is the regulation impermissibly vague or overbroad?

Third, is the regulation a prior restraint on speech?

Wagner: Tolerance isn’t with speech you disagree with, it’s with speech you abhor. We can (1) censure the speech; or (2) respond to it with more speech. Free speech only works if it is free without consequences.

KEY POINTS

Content neutrality

Wagner: The difference between a content-based law passed by government and a content-neutral law is the level of scrutiny. We don’t want to give the government power to restrict a person’s speech or expression because of what is in the message or the viewpoint of the person speaking.

Government cannot control the content of a private speaker’s message.

Turner Broadcasting v. FCC (1994 and 1997) Strict Scrutiny for Content-based regulations and Intermediate Scrutiny for Content-neutral based regulations. Federal law can require cable television systems to carry certain programming. The Court affirmed the opinion of the District Court that there was substantial evidence supporting the judgment of Congress that the must-carry provisions further the following important government interests: (1) preserving the benefits of free, over-the-air local broadcast television; (2) promoting the dissemination of information from a multiplicity of sources; and (3) promoting fair competition in the television programming market. The Court also ruled that the must-carry provisions do not burden substantially more speech than is necessary to achieve the government’s interest.

Has a regulation been adopted because government favors or disfavors the message?

Two concepts:

1. Subject matter neutrality – the Iraq war; abortion

2. Viewpoint neutrality – the war is good or bad; abortion is good or bad

Wagner: Being able to have the freedom of who we are through our expression is one of the reasons the Court will to go to as a justification for elevating this to a fundamental right.

Vagueness and Overbreadth

A speech regulation that provides no ascertainable standard of conduct is unconstitutionally vague

Coates v. Cincinnati (1971) See below.

If a speech regulation is substantially overbroad, a party whose speech or expressive conduct would otherwise be unprotected may raise a challenge.

Schad v. Borough of Mt. Ephraim (1981) Convictions, pursuant to zoning ordinance prohibiting live entertainment, of operators of adult bookstore featuring live nude dancing, held invalid under First and Fourteenth Amendments.

Distinct but often related concepts

Prior restraints

Certain efforts by the government to suppress speech before it happens are categorized as prior restraints, and are presumptively invalid

Most prior restraints come in the form of :

An administrative license or permit scheme that forbids speech in advance of the time it is to occur; or

A court order forbidding speech, on penalty of contempt

CONTENT NEUTRALITY

Wagner: If the government is regulating the subject matter or the viewpoint, it’s going to be content based. If the government is not regulating subject matter or viewpoint but is, in a neutral way, says no demonstrations in the park, it is content neutral. Strict scrutiny for content based regulation. Intermediate scrutiny for non-content based regulation.


The Distinction between Content-based and Content-neutral Laws

Content-based regulations are presumptively invalid.

Turner Broadcasting v. FCC. (1994) (1054 text)

First Amendment does not allow government to control content of private speaker’s message

Strict scrutiny applied to laws that “suppress, disadvantage, or impose differential burdens” based on content

Laws that compel speaker to deliver a particular message also get strictly scrutinized

Content-based restrictions on speech must meet strict scrutiny

Content-neutral regulation only need meet intermediate scrutiny

CONTENT NEUTRALITY


The Distinction between Content-based and Content-neutral Laws

A law is content-neutral if it applies to all speech

e.g, no demonstrations in park is content neutral

A law is content-based if the regulation turns on content of speech

e.g., no demonstrations on a specific topic or specific viewpoint is content based

How to determine whether a law is content based:

The requirement that the government be content-neutral in its regulation of speech means the government must be:

Viewpoint neutral; and subject matter neutral.

Viewpoint neutral means government cannot regulate speech based on ideology of the message.

Subject matter neutral means government cannot regulate speech based on the topic of the speech.

A law regulating speech is content-neutral if it applies to all speech regardless of the message

A law is content-based, therefore, if it is either a subject-matter or a viewpoint restriction

Boos v Berry (1988) (1058 text) The District of Columbia cannot prohibit signs near embassies just because the signs might denigrate the foreign countries which operate the embassies.

Republican Party of MN v. White (2002) (1060 text) It is an unconstitutional content-based restriction on political speech for a state judicial conduct rule to prohibit candidates for judicial office from announcing their views on disputed legal or political issues.

Wagner: The purported state interest Minnesota has in passing this regulation is impartiality – they don’t want people running for office to talk about disputed issues. This is a subject matter restriction and the regulation is struck down.

It’s important to look at the law and if you want it struck down, you are going to try and characterize the regulation that is limiting speech as content based because you have a much higher level of scrutiny. That’s what it comes down to – how the Court characterizes the issue.

Strip club example: If it looks and smells like content based regulation, but you have facts showing that the government’s reason was content neutral, then perhaps the government can convince the Court to call it a content neutral regulation. The intent may not be to restrict speech but to protect society from the collateral damage of drug dealing, robberies, and prostitution – activities that seem to follow areas with strip clubs.

Abortion clinic example: A statute says, “No protests outside an abortion clinic.” It sounds like content based because if it’s a protest in front of an abortion clinic, you are protesting abortion. If you are the government lawyer trying to defend the statute, you would argue that you are protecting privacy or some other secondary effect. The Court upheld the statute.

The government can make content based choices as long as the government is viewpoint neutral. If the government is the one running the library, the government can decide which books to have and make content-based choices as long as they are viewpoint neutral.

VAGUENESS AND OVERBREADTH

Wagner: If you see a government regulation and it looks vague or overbroad, make sure you stop at both islands because not all the time, but just as a matter of exam strategy or constitutional law strategy, if it’s one of them, there’s a good chance that it’s the other as well.

Unconstitutional Vagueness

A law is unconstitutionally vague if a reasonable person cannot tell what speech is prohibited and what is permitted.

Unduly vague laws violate due process whether or not speech is regulated.

Speech regulations may have a chilling effect even on protected expression

Also concerned with discriminatory and arbitrary enforcement

Issue is whether the regulation enables a reasonable person to distinguish between that expression which is prohibited and that which is allowed

Coates v. City of Cincinnati (1971)(1085 text) It violates principles of vagueness and overbreadth for a city to criminalize merely congregating on a sidewalk in a manner annoying to passerby. The Court said this statute was constitutionally defective because it provided no ascertainable standard of prohibited conduct, and because it could be used to punish protected exercises of the right to assembly.

Wagner: If you are picked up for loitering, you are going to argue the First Amendment – why? because it’s too vague. It could depend on the cop and what kind of day he’s had (no coffee or donut).

Chicago v. Morales (1999) It is unconstitutionally vague for an ordinance to require a police officer to order a person whom he reasonably believes to be a gang member to leave the area.

Wagner: Selective prosecution – too much discretion for the government. The police officer can pick and choose who is in the wrong place and it also violates procedural due process by not giving notice to people to whom it might be applied. The underlying concern is that it will chill expression. Because a vague law cannot determine what is to be prohibited, everybody will be afraid to do anything for fear that discretion will be selectively prosecuted. For this type of statute, use the vagueness doctrine.

When the government regulates speech, the Court is pretty clear – when the government passes a law that regulates speech, it has to do so with sufficient precision that a reasonable person can understand what is regulated so that we have notice and know whether the conduct is prohibited or permitted. The Court is going to strike down a statute that doesn’t do that.

Unconstitutional Overbreadth

A law is unconstitutionally overbroad, if

it regulates substantially more speech than the Constitution allows to be regulated, and

a person to whom the law constitutionally can be applied can argue that it would be unconstitutional as applied to others.

Even where government can regulate speech, (e.g., obscenity), a law restricting substantially more expression than Constitution allows can be declared unconstitutional on overbreadth grounds.

Because of importance of free speech, the Supreme Court makes exception to the general rule prohibiting third- party standing to challenge constitutionality of statute as applied to others.

Wagner: Fire and Corvette example: The water was necessary to put out the fire, but was it necessary to spray the nearby car?

Throw water on student example: Throw water on one student and it gets on the students in the front row.

An overbroad law is written in a way that covers more speech than is necessary.

The overbreadth doctrine can strike down a statute limiting government power An overbroad statute can be struck down because speech is so important – it’s a fundamental right.

Because speech is so important, we will allow third party standing.


Substantial Overbreadth Required to Challenge Law on Overbreadth Grounds

If a statute regulating speech is substantially overbroad, à a party whose conduct would otherwise be unprotected may raise a challenge.

Broadrick v. Oklahoma (1973) The Court rejected the overbreadth challenge saying that especially when conduct and not merely speech is involved, the Court will entertain an overbreadth challenge only when the overbreadth is real and substantial in relation to the plainly legitimate sweep of the statute.

What is substantial overbreadth?

Merely being able to conceive of a few impermissible applications not enough. City Council v. Taxpayers for Vincent (1984)

Must be able to show a significant number of situations where regulation would bar protected speech. Houston v. Hill (1987) (ordinance was “susceptible of regular application to protected expression.”)

Contrast New York v. Ferber (upholding state law prohibiting child pornography, where the law would apply to relatively few situations involving constitutionally protected speech)

Wagner: With the facts, look at the number of situations where the law could be applied to cover and prohibit constitutionally protected speech. If there are a lot of situations where the law could be read to cover constitutionally protected speech, then there is a better chance that the Court will say that it is substantially overbroad.

Child pornography is unprotected speech. If a statute is written protecting child pornography, it doesn’t get strict scrutiny because it is unprotected.

If a child pornography statute was overbroad, a child pornographer may argue that, yeah I’m bad, but this statute applies not only to me but it’s written in such a broad way that it is interfering with other people’s constitutional rights. I’m going to assert their rights now and seek to have the statute struck down as overbroad.

Exception to the General Prohibition Against Third Party Standing

An individual whose speech is unprotected by the First Amendment and who could constitutionally be punished under a more narrow statute may argue that the law is unconstitutional because of how it might be applied to third parties not before the Court.

Schad v. Borough of Mt. Emphraim (1981) (1087 text)

Overbreadth – strong medicine

Strong medicine: Striking down the statute on its face because of the way it’s written before the case even comes before the Court.

Overbreadth doctrine is “strong medicine,” because:

it involves the facial invalidation of a law; and

it permits individuals standing to raise the claims of others not before the Court.

The Supreme Court avoids invalidating laws on overbreadth grounds – two strategies:

1. By allowing courts to construe statues narrowly and thus avoid overbreadth Look to see if it’s a state law and if it’s overbroad, before it goes to the Supreme Court, the state supreme court construed the statute in a narrow way. The Supreme Court will sometimes accept the narrow construction from the state supreme court to uphold the law and avoid striking it down on overbreadth.

OR

2. By severing unconstitutionally overbroad part of the law from the remainder of the statute. Look at the whole statute and redact or sever the unconstitutional part – the Court will “strikeout” unconstitutional provisions then uphold the statute as being unconstitutional.

DISTINCT CONCEPTS

Vagueness and overbreadth frequently go together, but need not be so.

Coates, supra, involved a law that was both vague (no ascertainable standard) and overbroad (punished protected speech)

Board of Airport Commissioners v. Jews for Jesus, Inc. (1987)

Ordinance banning First Amendment activities at airport was overbroad, but not vague.

PRIOR RESTRAINTS


What is a Prior Restraint?

Defined: an administrative system (license or permit requirement) or judicial order (injunction) that prevents speech from occurring. Alexander v. United States (1993)

This definition is incomplete; other types of speech regulation may also be prior restraint.

Example: Seizing copies of newspapers or other materials; but see Alexander v. United States (1993) (no prior restraint where government seized and destroyed inventory of non-obscene videos)

Wagner: If you have a prior constraint, you are allowing the government to control what you are saying. It would be chilling expression before it ever takes place. No one will take the chance of speaking if someone knows he or she will be punished for it.

Why are Prior Restraints Bad?

“A free society prefers to punish the few who abuse rights of speech after they break the law than to throttle them and all others beforehand.”

Prior restraints on speech:

Bring a huge amount of expression under government oversight;

Chill and prevent communication before it takes place;

Permit little opportunity for the public to review and criticize

Courts are reluctant to find a law or order a prior restraint since so strongly disfavored

The collateral bar rule makes prior restraints especially harsh:

“a court order must be obeyed until it is set aside.

... persons subject to the order who disobey it may not defend against the ensuring charge of criminal contempt on the ground that the order was erroneous or even unconstitutional.”

When are Prior Restraints Permissible?

Court orders

Near v. Minnesota (1931) (judicial orders preventing speech constitute a prior restraint). The Court invalidated, as a prior restraint, the application of an ordinance that authorized a court to perpetually enjoin the publication of a newspaper of any malicious, scandalous, or defamatory material. The Court said that prior restraints may be justified only in exceptional cases, such as to prevent obstruction of the nation’s recruiting service, or to prevent the publication of the location of troops during wartime.

“The object…is not punishment…but suppression.”

“This is the essence of censorship.”

Proper remedy for false accusations is subsequent punishment under, for example, libel laws

What can be restrained?

  • So-called “troop ship” national security exception
  • Obscene materials
  • Incitement to illegal activity and government overthrow

A court order regulating speech must be narrowly tailored; Any injunction restricting speech must “burden no more speech than necessary’ to accomplish its objective.”

An injunction ‘issued in the area of First Amendment rights must be couched in the narrowest terms that will accomplish the pin-pointed objective permitted by constitutional mandate and the essential needs of the public order.”

Wagner: If you have a prior restraint, the government must narrowly tailor. Example: If you have a court order entering an injunction restricting speech, the speech must meet strict scrutiny. It has to burden no more speech than is necessary to accomplish the objective.

National Security as Reason for Court Order Stopping Speech

New York Times v. United States (1971) (1097 text) The Court refused to enjoin newspapers from publishing a classified study of the history of US decision making in Vietnam. In a per curiam opinion, the Court said that any system of prior restraints comes to the Court with a heavy presumption against its constitutional validity, and that the government has a heavy burden of justifying the judicial enforcement of such a prior restraint.

Justices Black and Douglas took apparently absolutist views against prior restraints

Justice Brennan argued for strict scrutiny

Justice White noted lack of statutory authority to issue injunction and noted possibility of criminal sanctions

Justice Stewart said President had authority but had failed to meet burden

Gag Orders on Press to Protect a Defendant’s Right to a Fair Trial – three requirements Court must meet to justify:

First, there has to be a showing of extensive publicity without a prior restraint that will jeopardize the ability to select a fair and impartial jury.

Second, The Court must determine that ‘measures short of an order restraining all publication [would not] have insured the defendant a fair trial.” (CNN is going to publish it anyway example).

Finally, even if the first two requirements are met, a prior restraint is permissible only if it is determined that one would be a workable and effective method of securing a fair trial.

Nebraska Press Association v. Stuart (1976) (1104 text) – Prior Restraints to Ensure Fairness of Trials. Rule: Trial judges may not use prior restraints on the media to ensure the fairness of a trial if there are less drastic means available. The Court struck down a state court pre-trial restraining order which prohibited the media from publishing any confessions made by the defendant (unless made to the press), and any other facts “strongly implicative” of the defendant’s guilt. The state judge had also restrained the publication of any testimony or evidence given at the preliminary hearing. Using prior restraint analysis, the Court concluded that the trial judge had less drastic means available to preserve the defendant’s right to a fair trial, such as a change of venue, postponing the trial, or sequestration of jurors.

PRIOR RESTRAINTS
When are Prior Restraints Permissible?

Court Orders Enjoining Obscenity

Since obscenity is unprotected speech, prior restraint is justified

Paris Adult Theatre I v. Slaton (1973) Rule: The government may prohibit the showing of obscene films in a movie theater restricted to consulting adults. The Court upheld a state court’s injunction against the showing of obscene films in a movie theater restricted to consenting adults. The Court distinguished the Stanley case [§748] by saying that the privacy of the home that was controlling in Stanley was not present in the commercial exhibition of obscene movies in a theater.

Court Orders Seizing Assets of Illegal Businesses Convicted of Obscenity Violations

The government’s use of RICO law to seize assets of a store selling adult books and movies after its conviction of obscenity violations did not constitute a prior restraint.

Alexander v. US, 509 U.S. 544 (1993 ) (1109 text) PROCEDURAL POSTURE: Petitioner criminal filed an application for a writ of certiorari to the United States Court of Appeals for the Eight Circuit to challenge the district court's forfeiture of his adult entertainment business as punishment for violation of the Racketeer Influenced and Corrupt Organizations Act.

OVERVIEW: Petitioner criminal was convicted of violating obscenity laws and of violating the Racketeer Influenced and Corrupt Organizations (RICO) Act. In addition to a prison term, the district court ordered petitioner to forfeit certain assets that were directly related to his racketeering activity. Petitioner challenged the forfeiture, arguing that it violated U.S. Const. amendment I because the forfeiture effectively shut down his adult entertainment business and constituted a complete ban on his future expression of free speech. Petitioner also argued that the forfeiture violated U.S. Const. amendment VIII as an excessive form of punishment. The court held that the forfeiture did not constitute a "prior restraint" under U.S. Const. amendment I because it did not forbid petitioner from engaging in any expressive activities in the future. The court found, however, that the appellate court failed to determine whether the forfeiture was excessive under U.S. Const. amendment VIII. Thus, the court rejected the claim under U.S. Const. amendment I, vacated the judgment, and remanded the claim for a determination of whether the penalty was excessive under U.S. Const. amendment VIII.


OUTCOME: The court vacated and remanded the appellate court's affirmance of the district court's forfeiture penalty against petitioner criminal for violating the Racketeer Influenced and Corrupt Organizations Act. The court held that although the forfeiture of petitioner's adult entertainment business did not violate U.S. Const. amend I as a prior restraint, a determination of whether the penalty was excessive was needed.

Wagner: Big Picture Review – Everything we’ve talked about so far, the government has already acted by passing a law that is affecting someone or some government official is engaging in some type of action that is affecting someone. The first step you always want to look at is, did the state or Congress have the authority to do that in the first place? Second, did they act within the scope of that authority? If the answer to either one of those questions is no, then you don’t even get to the limits on the Constitution that we’ve been talking about with regard to due process and the First Amendment.

Assuming the state or government is acting within its scope of power (enumerated in the Constitution or otherwise), then the question becomes, does some other part of the Constitution limit that government power? Does the free speech clause limit the government?

For free speech: I can’t give you a step by step roadmap, apply the facts, and hand in your paper. Each one of the doctrines under the First Amendment Speech Clause is going to either apply or not apply in any given combination depending on what your fact are. Look at it as a cruise ship going to each island.

Prior restraints are exactly what the framers did have in mind when they wrote the First Amendment because that’s what they were dealing with in England. The two most common prior restraints today are: (1) the court ordered injunction and gag order for the press; and (2) having to go to the government for permission to speak.

Most of the time when you see a prior restraint, the presumption is that the Court is probably going to strike it down. What you want to know is when the Court is likely to uphold it or when is a prior restraint through a licensing system going to be upheld.

First, the government has to have an important reason for the license or permit requirement.

Example: If you are running a city, you want to make sure that there aren’t two parades at the same time, you have a system that insures that two parades don’t happen at the same time.

Second, are there clear standards in the licensing or permit scheme?

The standards have to be very, very clear. The key is most cases will fall or not fall on discretion – it must leave no discretion to the government.

Example: the permit giver in some city hall office doesn’t get to be the king of speech for the city. The permit process has to afford almost no discretion.

Third, there has to be procedural place guards to ensure a prompt decision and judicial review.

The burden of proof is on the government. There has to be a system set up so that when the dispute comes up it can be resolved very quickly. Waiting a year for the decision is the same as censuring the person from talking. You can’t have an ex parte proceeding – it has to be an adversarial proceeding. Speech is a fundamental right; therefore, we are going to have pretty significant procedural safeguards to protect that fundamental right. It sounds like procedural due process

PRIOR RESTRAINTS
When are Prior Restraints Permissible?

Licensing and permits: Court has announced three requirements for validity

First, the government must have an important reason for the license or permit requirement

Cox v. New Hampshire (1941)

City said permit scheme was needed so city would have notice of parades or demonstrations, allowing it to provide proper policing and ensure only one parade or demonstration at a particular place and time

Court agreed this was an important reason

Licensing and permits: Three requirements cont’d

Second, the standards for granting or denying license or permit must be clear and must afford almost no discretion to government

City of Lakewood v. Plain Dealer Publishing Co (1988) (1118 text) Unbridled licensing discretion allows facial challenge without need for permit application and denial

  • Unfettered discretion risks self-censorship

  • Absence of standards makes it difficult to distinguish legitimate from illegitimate permit denials

Third, must be procedural safeguards in place to assure prompt decision and judicial review

Leading case is Freedman v. Maryland (1965). A licensing scheme for adult businesses will be constitutional if the government carries the burden of proving to a court that the work in question is obscene.

Burden of proof on government

Must be a prompt decision whether license will be granted

Must be an adversarial hearing; ex parte proceedings insufficient

Prompt judicial review must be available for any license denial

Freedman standards have been applied in other situations:

Postal stop orders for obscene materials. Blount v. Rizzi (1971)

Customs seizures of obscene materials. United States v. Thirty-Seven Photographs (1971)

One year before final judicial resolution is too long for injunction to remain in place.

National Socialist Party of America v. Village of Skokie (1977)

PRIOR RESTRAINTS

Licensing and permits: illustrative cases

Watchtower Bible and Tract Soc. v. Village of Stratton (2002) A municipal ordinance that makes it illegal for uninvited canvassers to go on private residential property to promote any cause, without first obtaining a permit, violates the First Amendment. Had the ordinance been applied only to commercial activities and the solicitation of funds, it might have withstood constitutional scrutiny as an acceptable means to achieve the city’s interests in protecting resident’s privacy and preventing fraud. The requirement of signing the registration form also compromised the anonymity to which speakers are entitled under McIntyre v. Ohio Election Commission.

Preventing fraud and crime and protection of residents’ privacy are important government interests

But Justice Stevens did not speak to the issues of discretion and procedural safeguards

Said permit requirement was overbroad

Not appropriately tailored to meet stated government interests

Licensing and permits: illustrative cases

Watchtower Bible, cont’d

Chief Justice Rehnquist, dissenting, criticized the majority:

Contrary to a long line of precedent allowing permits for residential canvassers

Ordinance did not prohibit canvassing; merely required a permit

No discretion vested in mayor to deny permit

Would have reviewed under intermediate scrutiny applied to time, place, or manner regulations

Licensing and permits: illustrative cases

City of Littleton v. Z.J. Gifts D-4, L.L.C. (2004)

Prompt judicial review includes prompt judicial decision

How prompt must prompt judicial review be? No clear answer

Colorado’s ordinary judicial review procedures suffice as long as the courts remain sensitive to the need to prevent First Amendment harms and administer those procedures accordingly.”

Question for case-by-case, rather than a facial, challenge

Licensing and permits: illustrative cases

City of Littleton, cont’d

Rationale:

State’s judicial review procedures provide means to avoid First Amendment delay-related harms

Presume state’s judges willing to exercise powers to avoid delay-related harms

Ordinance here did not seek to censor; criteria for license were objective, nondiscretionary, and unrelated to content

Incorporation of judicial review safeguards into local laws is a matter for the state to decide

What is an infringement of speech?

First Amendment analysis only triggered if there is an infringement

A direct and substantial interference is an infringement.

law prohibiting speech is an infringement

prior restraints are an infringement

civil liability is infringement

precluding payment for books involving crimes are infringement

Government compelled speech is an infringement

unconstitutional conditions doctrine

Constitutional Law II

Assignment 7: Freedom of Expression


Types of Unprotected and Less Protected Speech

Ø Introduction into content-based unprotected speech

Ø Incitement to Illegal Conduct

Ø Fighting Words

Ø Obscenity and Child Pornography

Wagner: when should the government be able to prohibit or regulate free speech? The Court has said there is speech that is unprotected and speech that is less protected.

UNPROTECTED AND LESS PROTECTED SPEECH:

  • Incitement of illegal activity
  • Child pornography
  • Fighting words, and
  • Obscenity

INCITEMENT OF ILLEGAL ACTIVITY

BRANDENBURG TEST: SPEECH THAT INCITES IMMINENT LAWLESS ACTION AND IS LIKELY TO PRODUCE IT

Memorize Brandenburg Test for Exam

Under a clear and present danger analysis, the government tries to punish a speaker for inciting a listener to engage in illegal activity.

A conviction for incitement only is constitutional if several requirements are met:

1. Imminent harm;

2. A likelihood of producing illegal action; and

3. An intent to cause imminent illegality.

Brandenburg v. Ohio (1969) (1171 text) The Modern Test. Ask whether the speaker’s advocacy of illegal action is directed to inciting or producing imminent lawless action and is likely to incite or produce such action by the audience. The government may punish speech if it is directed to inciting imminent lawless action and is likely to cause such action. The Court has ruled that if speech is not directed to a specific person or group, it is not punishable under clear and present danger doctrine because it is not likely to produce any imminent lawless action.

Note: mere advocacy of the use of force or violence does not remove speech from the protection of the First Amendment.

Wagner: The Court in this case struggles with clear and present danger test. What factors should the Court consider when it is trying to figure out if there is a clear and present danger and therefore the speech be unprotected? Seriousness of the harm, imminence, and likelihood? Are they independent? Do you weigh one against the other or are they part of the same analysis? Is the harm so bad that the likelihood doesn’t matter? Brandenburg doesn’t answer the question but it does state the test.

Example: Use the Brandenburg Test when the government is regulating your illegal activity or you are advocating the overthrow of the government. Incitement is an island for unprotected speech, therefore apply Brandenburg Clear and Present Danger Test.

For the “A” answer: There is no incitement because there is no imminent action on the part of anybody. There is no imminent harm, no likelihood of producing an illegal action. He has bad and hateful ideas but there is a difference between prosecuting someone for their ideas and prosecuting someone because what they are saying is going to result in imminent harm and have a likelihood of producing action because it is directed at a particular person. Always explain “why”.



TRUE THREATS (related to Brandenburg but distinct)

No First Amendment protection for a “true threat.

Circuit split as to how to determine whether speech should be regarded as a “true threat.”

Some circuits say it’s determined from the perspective of the reasonable listener;

Other circuits say it’s assessed from the perspective of the reasonable speaker.

Truth doesn’t matter; it’s the risk of the words causing to react.

FIGHTING WORDS

Fighting Words: Scurrilous epithets, delivered in person to the listener, which are likely to cause an average addressee to retaliate against the speaker.

S & S: Fighting words, offensive speech, and hate speech all have one common thread: each involves nasty, scurrilous language that is intended or likely to offend some people or inflict psychic trauma on the members of the audience. The constitutional question is when does this kind of speech cross the line and become regulable by the government. Remember that preventing hurt feelings or psychic trauma is not a legitimate government interest. There must be more such as preventing a breach of the peace.

When may speech be punished because of the risk that it might provoke an audience into using illegal force against the speaker?

“Fighting words” are a category of speech unprotected by the First Amendment.

Chaplinsky v. New Hampshire (1942) (1174 text) (those words “which by their very utterance inflict injury or tend to incite an immediate breach of the peace.”). The fighting words doctrine requires a personally abusive epithet to be directed to the listener in such a manner as to provoke an average addressee to violence.

Wagner: if fighting words are unprotected speech, the question becomes, what are fighting words?

Words which “by their very utterance inflict injury or tend to incite an immediate breach of the peace.”

Chaplinsky is still good law, but Court has never since used it to uphold a conviction.

The Court has used 3 techniques to reverse convictions (w/o overturning Chaplinsky)

1. By narrowing fighting words doctrine;

2. By finding fighting words law unconstitutionally vague and overbroad; and

3. By finding fighting words law to contain an impermissible content-based speech restriction.

First Technique Used To Reverse Convictions (w/o overturning Chaplinsky): NARROWING THE DOCTRINE

Street v. New York, (“‘fighting words’ [are those few] which are likely to provoke the average person to retaliation, and thereby cause a breach of the peace.’”)

Cohen v. California, (unprotected fighting words occur only if the speech is directed to a specific person and likely to provoke violent response).

Texas v. Johnson: (no one could interpret flag burning as a direct personal insult or an “invitation to exchange fisticuffs.”)

Example: Burning a flag in the presence of Purple Heart awardees = fighting words. Burning a flag six blocks away from Purple Heart awardees = not fighting words. It has to be directed to a specific person that is likely to produce a violent response.

Second Technique Used To Reverse Convictions (w/o overturning Chaplinsky):
LAWS INVALIDATED AS VAGUE AND OVERBROAD

Since Chaplinsky, Court has reversed convictions involving fighting words by declaring the laws constitutionally vague and overbroad

Fighting words law upheld only if it is narrowly tailored to apply just to speech that is not protected by the First Amendment.

Gooding (1972) (1177 text)

Rosenfeld v. New Jersey

Lewis v. City of New Orleans

Brown v. Oklahoma

R.A.V. v. St. Paul (1992) The government cannot selectively ban certain categories of fighting words based on their content. In other words, the government cannot prohibit only those fighting words which it dislikes.

Wagner: it is regulating content within a category of unprotected speech. If the government regulates in a content-based way within the area of unprotected speech, the Court is going to apply strict scrutiny to the statute.

Third Technique Used To Reverse Convictions (w/o overturning Chaplinsky):
FINDING FIGHTING WORDS LAW TO CONTAIN AN IMPERMISSIBLE CONTENT-BASED SPEECH RESTRICTION

Content-based distinctions within a category of unprotected speech will have to meet strict scrutiny, subject to two exceptions.

1. A content-based distinction is permissible if it directly advances the reason why the category of speech is unprotected;

§ e.g., obscenity law could prohibit the most sexually explicit material without having to ban everything that is obscene.

2. A law will not be held content-based if directed at remedying secondary effects of speech justified without respect to content.

Wagner: Don’t forget – even though fighting words are unprotected speech and therefore can be regulated and you can then be prosecuted, the statute or regulation can be overbroad or vague and get struck down. If the government tries to re-write the statute to be more specific, it can be struck down for impermissible content based speech as in the R.A.V. case. Argue both sides.

Unprotected Speech - Fighting Words (continued)


HOSTILE AUDIENCE REACTION

When government may punish individuals for speech that provokes a hostile audience reaction.

Feiner v. New York (1951): “[W]hen as here the speaker passes the bounds of argument or persuasion and undertakes incitement to riot….”

Hostile audiences doctrine now narrowed

Gregory v. City of Chicago (1969): conviction overturned because ordinance not limited to threats of imminent violence

Wagner: Before you take the speaker away, you have to show that you tried to get the audience under control and show that there is a threat of imminent violence.

TRUE THREATS

Virginia v. Black (2003) A state may, consistent with the First Amendment, ban “true threats,” which include cross burning carried out with the intent to intimidate. True threats comprise a category of speech unprotected by the First Amendment. The Court also said that the speaker need not actually intend to carry out the threat.

  • First Amendment allows true threats to be banned

“[T]he speaker means to communicate a serious expression of an intent to commit an act of unlawful violence to a particular individual or group….”

Need not intend to carry out threat

Prohibition protects from the fear of violence

Intimidation (including cross burning) can be a true threat

  • A ban on cross burning with intent to intimidate would be consistent with holding in R.A.V.

But the law said that cross burning was, itself, prima facie evidence of intent to intimidate

  • This prima facie evidence provision rendered the statute unconstitutional

True that many cross burnings are done with specific intent to intimidate

But cross burning may have other symbolic or expressive purpose that cannot be proscribed

Might reach protected political speech

Wagner: This is why it is so important for you to frame the issue. If you can frame the issue as a true threat, it is much more likely that you are going to get the statute upheld as opposed to if you framed it as a fighting words statute because you are stuck with narrow construction, overbreadth, content based and the R.A.V. situation. An attorney for the government would argue true threat to uphold the statute where a defense attorney would argue fighting words.

SEXUALLY ORIENTED SPEECH

Unprotected Speech: Obscenity

Obscenity is a category of speech unprotected by the First Amendment.

Roth v. United States, (1957) (1203 text)

Paris Adult Theater I v. Slaton (1973) (1205 text)

Miller v. California (1973) (1208 text)

Three Elements - Miller v. California Test (memorize):

1. Whether average person applying contemporary community standards would find that work as a whole appeals to prurient interest

2. Whether work depicts or describes, in a patently offensive way, sexual conduct specifically defined by state law

3. Whether work taken as a whole lacks serious literary, artistic, scientific, or political value

v MATERIAL MUST MEET ALL THREE REQUIREMENTS FOR IT TO BE HELD OBSCENE.

First Element – prurient interest:

Material must appeal to the prurient interest for the average person, applying contemporary community standards

For purposes of obscenity, prurient means a “shameful” or “morbid” interest in sex (as compared to a “normal” interest) Brockett v. Spokane Arcades, Inc.

“Prurient interest” is determined by a community standard

Prurient interest is essentially a question of fact, and our nation is simply too big and too diverse” to expect a single national standard

Community standards judged by the average person, not the most sensitive or the most insensitive

Second Element – patently offensive sexual conduct

Miller gave an example of “[P]atently offensive ‘hard core’ sexual conduct…”-- said state can define a law as follows:

Patently offensive representations or descriptions of ultimate sexual acts, normal or perverted, actual or simulated;

Patently offensive representations or descriptions of masturbation, excretory functions, and lewd exhibition of the genitals.”

Patently Offensiveness is determined by a community standard

Patently offensive is essentially a question of fact, and our nation is simply too big and too diverse” to expect a single national standard

Community standards judged by the average person, not the most sensitive or the most insensitive

Third Element – lack serious redeeming value

To be obscene, the material must, taken as a whole, lack Serious, Literary, Artistic, Political, or Scientific value (SLAPS test).

Redeeming Value is to be determined by a national standard—how the work would be appraised across the country—and not a community standard.

“the value of [a] work [does not] vary from community to community. . . .

The proper inquiry [is] whether a reasonable person would find such value in the material.”

Pope v. Illinois (1987) The Court emphasized that the third prong of Miller (SLAPS test) is to be judged by a reasonable person standard, rather than by contemporary community standards. The idea is to provide a more uniform standard throughout the country, thereby reducing regional differences in First Amendment interpretation.

Wagner: This is a community standard versus a national standard.

Unprotected Speech: child pornography

New York v. Ferber (1982) (1213 text) The government may prohibit the sale of child pornography.

Prevention of sexual abuse and exploitation of children a state interest of “surpassing importance”

States have a compelling interest in prosecuting those who sexually exploit children

Child pornography, like obscenity, is unprotected

Child pornography does not have to fit under the Miller test for obscenity though in order to ban.

Wagner: The issue that will come up in a child pornography constitutional challenge is whether the state has adequately defined the conduct. You would have to frame it as pictures of children that include sexual conduct.

New York v. Ferber did not attempt to define child pornography; it simply upheld the definition contained in the New York law.

State law must adequately define conduct, including sexual conduct, to be prohibited

Offenses limited to works that “visually depict sexual conduct by children below a specified age.”

No criminal liability absent scienter

It is unprotected speech; therefore it only has to be rationally related to a legitimate government purpose. If the statute adequately defines the conduct, and limits to works that visually depict sexual conduct of children, the Court will uphold the statute.

Virtual Child Pornography

Child pornography must involve actual children; government cannot ban digitally created images or depictions of adults portraying minors. Ashcroft v. Free Speech Coalition (2002) (1216 text) because there is no actual sexual abuse of children involved in the making of the images

Private Possession of Child Pornography

The government may prohibit all possession of child pornography, even in the privacy of the home.

Osborne v. Ohio (1990) The Court upheld an Ohio statute that criminalized the private possession of child pornography. Osborne possessed photographs of a nude boy in a number of sexually explicit poses. Stanley v. Georgia (a state cannot criminalize the private possession of an obscenity) was distinguished on the basis of the state interest advanced in each case. In Stanley, the state was trying to prevent obscenity from poisoning the minds of the viewers; in Osborne, the state was trying to prevent sexual abuse and exploitation of children.

No constitutional right to possess child pornography in the home; compelling reasons exist to prohibit possession. Stanley distinguished. Osborne v. Ohio (1990)

Wagner: We prohibit drugs and distribution and because of the dangers, we prohibit the possession of drugs. We don’t prohibit the possession of obscenity in the home but we will prohibit the possession of child pornography. Prior restraints are therefore allowed as they are unprotected material.


Wagner: First step: Don’t forget about the “big picture” – the government needs to have a power source in order to pass the law in the first place. If the government doesn’t have a power source, or were acting outside the power source, you don’t even get to the question of whether or not it is commercial speech or whether or not the O’Brien test applies, or whether or not an “island” applies.

Second step: Does some other part of the Constitution limit that government action? When does the First Amendment speech clause limit government power? Less protected speech = more regulated speech.


Assignment 8: Freedom of Expression


Types of Unprotected and Less Protected Speech

Commercial Speech

Advertising Illegal Activities

False and Deceptive Advertising

Conduct the Communicates (Symbolic Speech)

LESS PROTECTED SPEECH

COMMERCIAL SPEECH


What is Commercial Speech?

No clear definition;

S & S: Commercial speech is speech that advertises a product or service usually for profit. To determine whether a restriction on commercial speech is permissible, first ask whether the commercial speech concerns a lawful activity, and whether it is misleading to consumers. If the commercial speech is not misleading, and is an advertisement for a lawful product or service, apply the following three-part test:

a) Does the government have a substantial (not compelling) reason for the regulation?

b) Does the restriction directly advance the government interest?

c) Is the restriction narrowly tailored to achieve the government interest?

Three characteristics:

1. It is an advertisement of some form;

2. It refers to a specific product, and

3. The speaker has an economic motivation for the speech.

Bolger v. Youngs Drug Products Corp. (1983) (1258 text) OVERVIEW: Appellee, manufacturer, seller and distributor of contraceptives, attempted to send unsolicited mailings to individuals concerning contraception and contraceptives. Appellant, United States Postal Services, prevented Appellee from sending the mailings, arguing that under 39 U.S.C.S. § 3001(e)(2), sending information concerning contraceptives through the mails was illegal. Appellee sued appellant for a declaratory judgment, on the basis that such a prohibition of a mailing was unconstitutional. The Court held that the mailing concerned commercial speech, which was generally afforded less constitutional protection than other forms of expression. The Court concluded that the law was unconstitutional under U.S. Const. amend. I, as a violation of Free Speech, since it unreasonably suppressed information concerning health issues, and there was no justification for preventing Appellee from sending such information to others.
OUTCOME: The Court affirmed the judgment of the lower court and held that federal law, which prevented the unsolicited mailing of information concerning contraceptives, was unconstitutional under the First Amendment right to free speech.

Wagner: If you have a product, and you are advertising it and your motive is profit related, then you have commercial speech.

Chinese toys damage control example: “Our toys are safe and we care about the children that chew on our toys.” Are they selling something specific? Is it commercial speech? I don’t know, we haven’t gone to court over it yet.

Does the Constitution Protect Commercial Speech?

Commercial speech is protected by the First Amendment

S & S: Up until the mid 1970s, commercial speech was not protected under the First Amendment, but was simply a kind of economic transaction which a government could regulate for any rational reason. Today, commercial speech is protected under the First Amendment, but receives only an intermediate level of protection.

Consumer’s right to receive information

Va. State Board of Pharmacy v. Va. Citizens Consumer Council, Inc. (1976) (1253 text). The Court invalidated a Virginia statute which prohibited the advertising of prescription drug prices by pharmacists. The law was challenged by consumers who asserted a right to receive information about prescription drugs prices. Speech does not lose First Amendment protection because money is spent to disseminate it, or because the advertiser has an economic interest in the advertisement. The consumer also has a strong interest in the free flow of information. The Court said that certain restrictions, such as time, place or manner regulations would be permissible, but it did not articulate any specific test by which to analyze commercial speech.

Wagner: This case solidifies the position of commercial speech in the less protected (but still protected) area of free speech. The interest is the free flow of marketplace of ideas. You can’t have a free marketplace without the free flow of ideas and information. It may not have the same value as “give me liberty or give me death” political speech, but it does have some value. It has to be truthful, non-misleading speech about lawful activity. Murder-for-hire is not going to receive protection as murder is illegal.

If the speech is determined to be commercial speech, the question becomes, can the government regulate it?

Kinds of laws regulating commercial speech

First, laws that outlaw advertising of illegal activities.

Such advertising is not protected by the First Amendment.

Second, laws prohibiting false and deceptive advertising.

Such ads are not protected by the First Amendment.

Third, laws prohibiting true advertising that inherently risks becoming false or deceptive.

Attorney advertising: “I’ve never lost a case” but has only had one case.

Fourth, laws that limit commercial advertising to achieve other goals

Enhancing the image of lawyers,

Decreasing consumption of alcohol or tobacco products,

Preventing panic selling of houses in neighborhoods,

Decreasing gambling.

Prevailing test in commercial speech

Central Hudson Gas & Electric v. Public Service Commission (1980) The Court has adopted a four-part test in determining whether commercial speech is entitled to constitutional protection.

Wagner: The government’s central interest in regulating advertising by the utility is to conserve energy. Also, the utility is a monopoly and therefore does not need to advertise in order to get people to use more than they need.

ANALYSIS FOR ANALYZING GOVERNMENT REGULATION OF COMMERCIAL SPEECH (Central Hudson Analysis):

1. To determine whether a law regulating commercial speech is permissible, first determine whether the commercial advertising concerns lawful activity and whether or not it is misleading.

Ask: Is the advertising truthful, not misleading, and for lawful activities?

(If the advertising false or deceptive, or of illegal activities, it’s unprotected by the 1st Am, and the government can regulate with rational basis review)

2. If the commercial speech is not misleading, and is for a lawful product or service, then determine whether the asserted governmental interest is substantial (sounds like intermediate scrutiny).

Ask: Does the government have substantial reason for the regulation?

3. If the government does have a substantial reason for the regulation then determine whether the regulation directly advances the governmental interest asserted,

Ask: Does the law directly advance the government’s interest?

4. If the government regulation directly advances the government’s interest, determine whether it is narrowly tailored to accomplish that interest.

Ask: Is the law narrowly tailored to achieve the government interest?

Advertising of Illegal Activity

Advertising of illegal activity is not protected by the First Amendment

Rational basis review of government action regulating advertising of illegality

(A billboard in New York promoting an online gambling site may have left the false impression among many that both the ads and online wagering are legal)

False and Deceptive Advertising

False and deceptive advertisements are unprotected by the First Amendment.

Rational basis review of government action regulating false or deceptive advertising

Wagner: The government can regulate lawyers preventing them from handing someone their card (solicitation) because there is not a level playing field and there is a risk of coercion that takes it outside the realm of protected speech. This also comes up with misleading trade names.


SYMBOLIC SPEECH: CONDUCT THAT COMMUNICATES

Symbolic speech is communication effected through conduct; there are no words involved.

Supreme Court long has protected conduct that communicates under the First Amendment.

When is conduct regarded as communicative?

Conduct is analyzed as speech under the First Amendment if,

there is the intent to convey a specific message; and

there is a substantial likelihood that the message would be understood by those receiving it

Spence v. Washington OVERVIEW: Defendant was convicted under Wash. Rev. Code § 9.86.020 for displaying a United States flag, which he owned, with a peace symbol fashioned with removable tape affixed to the flag. Defendant displayed the flag outside the window of his apartment. Defendant testified that he displayed the flag to protest the Vietnam War and the killings at Kent State University and to associate the flag with peace. The state appellate court reversed the conviction. However, the state supreme court reinstated the conviction. The United States Supreme Court reversed, holding that the statute was unconstitutional as applied to defendant. The Court noted that the flag was privately owned and was displayed on private property, that defendant engaged in no trespass or disorderly conduct, that the record was devoid of proof of any risk of breach of the peace, and that defendant engaged in a form of communication. There was no risk that defendant's acts would mislead viewers into assuming that the government endorsed his viewpoint. The Court concluded that no interest that the state could have had in preserving the physical integrity of a privately owned flag was significantly impaired under the facts.


OUTCOME: The court reversed defendant's conviction for improper exhibition of the United States flag.

When May Government Regulate Conduct That Communicates?

If it has an important interest unrelated to suppression of the message; and

If the impact on communication is no more than necessary to achieve the government’s purpose.

United States v. O’Brien (1968) (1316 text) (intermediate scrutiny for government regulation of conduct that communicates). The Court upheld a conviction for burning a draft card in violation of a federal law which made it a crime to knowingly destroy or mutilate a draft card. The Court ruled that when the government criminalizes symbolic speech (communication having both speech and non-speech elements), the law will be upheld if it passes the following test:

O’Brien Test

1. Is the law within a constitutional power of the government?

2. Does the law further an important or substantial government interest?

3. Is the government interest unrelated to the suppression of free expression? (Is it content neutral?)

4. Is the incidental restriction on speech no greater than essential to furthering the government interest? (Are the means narrowly tailored to achieve the government goal?)

Applying the O’Brien test -- Example: Flag Desecration

If the Court finds that the state is regulating communicative conduct (i.e., the regulation is not related to expression), then the less stringent O’Brien intermediate scrutiny standard for regulating non-communicative conduct controls –

But if instead the Court finds that the regulation is related to expression because of its content – then the court applies strict scrutiny to the content-based regulation

Texas v. Johnson (1989) (1320 text) (O’Brien intermediate scrutiny test NOT applied; Strict scrutiny applied) The Court refused to use the O’Brien test because the Texas law was content based (respectful burning of old flags is permissible, disrespectful burning is not). Content based restrictions receive strict scrutiny and the Texas law failed that standard because criminalizing flag burning was not narrowly tailored to achieve the State’s interest of preserving the flag as an unalloyed symbol of our country.

Wagner: The reason why strict scrutiny was applied and not intermediate scrutiny was because they had good lawyers that convinced the Court to frame the question of not whether or not this conduct that communicates but whether this is a content based regulation or not.

If we are on the speech island, the first question we have to ask ourselves is, is it symbolic speech? If it is symbolic speech then the second question is, can the government regulate it? If so, when?

If you get the Court to frame the issue as communicative conduct as symbolic speech, then you are going to apply O’Brien test. The Court in Texas v. Johnson uses strict scrutiny because they say the government is regulating the content of the message. They don’t say the government is regulating the non-communicative conduct, they say the government is regulating the content of the message. If you regulate the content of the message, you apply strict scrutiny.

The Court distinguishes O’Brien because in O’Brien the government interest was regulating conduct. Here, you can’t say that. The government is looking to suppress the (anti-American) message. You can use the flag for patriotic purposes but not for protest purposes – regulating based on its content. Currently there is a push to re-word the statute.

If the Court finds the issue as regulating communicative conduct (not related to expression), then you apply O’Brien or intermediate scrutiny. On the other hand, if the Court frames the issue and says that the government regulation is related to expression because of its content, then you apply strict scrutiny to a content based regulation (Texas v. Johnson).

Exam Question: Apply the O’Brien test to a statute that prohibits public nudity. You are the lawyer for the city (or the cabaret). Applying O’Brien, how does it come out?

Applying the O’Brien test -- Example: Nude Dancing

Barnes v. Glen Theatre, Inc. A generally applicable public nudity statute may be applied to prohibit nude dancing.

Court characterizes nude dancing as conduct that communicates; the Court expressly applied the O’Brien test.

The Court found that it is within a state’s police power to prohibit public nudity,

that there is an important interest unrelated to suppression of the message because of the state’s interest in morality,

and that the impact on communication was no greater than necessary because the dancers could still express their message, albeit with clothes.

Wagner: The Court said that while it is conduct that communicates it doesn’t survive the O’Brien test because the government has some substantial reasons that are unrelated to the suppression of the speech (state police power and health, safety, and welfare).

Spending Money as Political Speech

Political Speech is the Core of the First Amendment

Thus, restrictions on political speech are subjected to strict scrutiny

Spending money in connection with political campaigns is a form of speech protected by the First Amendment

expenditures of money here are not the same as conduct that communicates (therefore apply strict scrutiny and NOT the lower O’Brien standard)

Buckley v. Valeo The federal government may place limits on contributions to candidates for federal office. Using a First Amendment analysis, the Court said that the $1,000 limitation on expenditures for a candidate substantially restrains the quality and diversity of political speech, whereas the same limit on contributions to a candidate involves little direct restraint on political expression, and is necessary to achieve to government interest in preventing corruption and the appearance of corruption.

Note distinction between laws imposing contribution limits and laws imposing expenditure limits

The Court generally upholds laws imposing contribution limits and strikes down laws imposing expenditure limits. WHY:

Contributions to candidates create a greater danger of corruption and the appearance of corruption than expenditures.

Expenditures (taking your own money and spending it on campaigns) are much more directly related to speech than contributions.

Wagner: Contributing large amounts of money gives the appearance of potential corruption. What’s the difference? Does the limiting of money limit speech?

INTRODUCTION INTO TIME, PLACE, AND
MANNER RESTRICTIONS

Wagner: Ask yourself, is the government trying to regulate my speech entirely or is it just moving speech around?

If government is just restricting the time, place, or manner of speech (and is not completely prohibiting it), use the following four-part analysis/requirements:

ASK:

Is the restriction content-neutral?

Is it directed at achieving a significant government interest?

Is it narrowly tailored to achieve that interest?

Does it leave open ample alternative channels of communication? (without this last requirement you almost have a prior restraint and would have to apply strict scrutiny)

§ NOTE: When no ample alternative channels of communication exist, the law is really a prior restraint – to which a court applies strict scrutiny

Time, Place or Manner Restrictions on Conduct that Communicates (symbolic speech)

Time, Place or Manner Restrictions in a Public Forum (next week)

Wagner: What if the government says you can have nude dancing, but you can only have it 10 miles outside the car dealership. The government is moving the speech. If the government isn’t prohibiting the speech but rather moving it around, apply the time, place and manner regulation.

S & S: The Court has said that a time, place, or manner restriction imposed by an injunctive order is to be subject to more stringent review than one effected by a general ordinance. Because the injunction is directed at an actual or threatened violation of a legislative or judicial decree (as opposed to an ordinance, which is directed at an entire class of potential violators), it carries greater risks of censorship and discrimination. The test for a content-neutral injunction is whether it burdens no more speech than is necessary to serve a significant government interest.

CONSTITUTIONAL LAW II


ASSIGNMENT 9 - FREEDOM OF EXPRESSION

What Places are Available for Speech? When can the government limit the ability of speech?

Wagner: Be aware of state action when we are talking about forums because what you may see is someone being abused and you want to do something for that person but it is a private company acting.

We get our word out by protesting. The sidewalk is the poor person’s printing press and allows for a level playing field. Certain kinds of public property should be presumptively available for speech purposes.

Government Property and Speech I – Public Forums

Government Property and Speech II – Designated Public Forums

Private Property and Speech

Speech in Authoritarian Places

Speech Rights of Government Employees at Work

GOVERNMENT PROPERTIES AND SPEECH

After initially refusing to recognize any First Amendment right to use government property for speech purposes -- the Court now recognizes a right to use some government property under some circumstances for speech

Government power to regulate in public places

Hague v. CIO (not in assigned reading). The Court rejected the argument that a government’s ownership of property confers on it the same power to exclude speakers as that enjoyed by the owner of private property. Such use of the streets and public places has, from ancient times, been part of the privileges, immunities, rights, and liberties of citizens.

Schneider v. NJ (1939) (not in assigned reading) A city may not totally prohibit handbilling on public streets. The government may regulate the conduct of those using the streets, so long as their right to speak or distribute literature is not abridged. The city’s interest in preventing litter was not sufficiently important to justify a total ban on distributing handbills; less drastic means, such as punishing litterers, were available.



What government property must be made available for speech and under what circumstances?

Ask, what’s the nature of the government property? Then look at the regulation or statute. If you look at it carefully, you will see what kind of property it is. Depending on what kind of property it is will determine which rule to apply. In certain situations the government will have more authority to regulate and keeping out speech.

Three different types of government property, each with different rules for when government can regulate:

public forums: places where the government has the constitutional duty to allow speech to occur (streets, parks, etc. – places traditionally reserved for public expression)

limited public forums (a.k.a. designated public forums): places where the government voluntarily opens to speech (a place not generally open to the public); and

non-public forums: places where government can and does exclude speech (property the state uses for government purposes).

Access to School Mailbox Systems

Perry Education Assoc v. Perry Local Educators’ Assoc (1983) (1346 text). A school board may grant selective access to a school mailbox system. If a union is not a member of the official school community, a school may exclude it. The Court found that the losing teacher’s union was being excluded because of status (not being a member of the official school community) rather than any message it might convey.

Wagner: The issue becomes: what kind of forum is this? Perry is the case that decides what the forums are (see above).

Analytical Strategy: First determine which kind of forum exists and then apply the applicable rules.

Wagner: The main point of the Perry case is the test you apply and the constitutionality of the regulation regulating the speech is going to depend on the place and the nature of the government action – where the property is and what the purpose of the property is. You have to decide what type of forum it is. The lawyers for Perry are going to argue limited public forum – the school opened the mailboxes up – they are allowing other groups to use the mailboxes and once you open it up, you cannot exclude or regulate based on content. The lawyers for the school argued that it was a non-public forum and the Court decided in their favor for the way they framed the issue.

PUBLIC FORUMS

Public forums are government-owned properties that the government is constitutionally obligated to make available for speech.

e.g., sidewalks and parks

Public forums – when the government may regulate

1) The regulation must be content-neutral (unless the content-restriction is justified by strict scrutiny)

Police Dept of Chicago v. Mosley (1972) (1348 text) A city cannot selectively prohibit picketing based on the message on the picket signs. The Court invalidated a Chicago ordinance prohibiting picketing within 150 feet of a school, with the exception of peaceful picketing of any school involved in a labor dispute. The Court said this ordinance was not content neutral, and therefore was not to be analyzed under time, place, or manner rules.

2) The regulation in a public forum must be a reasonable time, place, or manner restriction that:

  • Serves an important government interest; and

  • Leaves open adequate alternative places for speech.

Hill v. Colorado (2000) (1351 text) The Court upheld a Colorado statue that regulates speech-related conduct within 100 feet of the entrance to any health care facility. The statute makes it unlawful within the regulated areas for any person to “knowingly approach” within eight feet of another person, without that person’s consent for the purpose of leafleting, displaying a sign, or speaking with that person. The statute was challenged facially by a group of anti-abortion protesters who wished to confront women outside abortion clinics. The Court upheld the statute as a content neutral time, place, or manner restriction.

3) A licensing or permit system for the use of public forums must:

  • Serve an important purpose;

  • Give clear criteria to licensing authority that leaves almost no discretion; and

  • Provide procedural safeguards such as requirement for prompt determination of license request and judicial review of license denials.

4) Finally, the Court has ruled government regulation of speech in public forums need not use the least restrictive alternative,

– Although it must be narrowly tailored to achieve the government’s purpose.

Wagner: If the government is regulating in a public forum, the first thing you should ask is, is the government regulating in a content-neutral way. The government cannot prohibit speech completely in a public forum and if the government tries to regulate speech in a public forum, it has to do it in a content neutral way.

Time, place, and manner and content-based restriction hypo: No campaigning within 100 feet of a polling place. This is one case where it did survive strict scrutiny and the Court upheld the law. It would intimidate voters or coerce them not to vote.

Passing out flyers at a fair hypo: The government’s interest is regulating crowd control. In order to have a valid time, place, and manner regulation, the statute must provide an adequate alternative place for the speech, e.g. using booths as a place for speech.

P 1359 5th Paragraph (Ward v. Rock Against Racism): The requirement of narrow tailoring is satisfied so long as the regulation promotes a substantial government interest that would be achieved less effectively absent the regulation. In order to play in Central Park, you must use city sound equipment and engineers and not your own. The government says it’s content neutral and the important government interest is noise reduction. The lawyer for the band argues that if it’s the loundness you are concerned about, regulate that and not the music engineers. There is a more narrow or restrictive way of tailoring this means to the objective. You don’t have to use the most restrictive means, it just has to be narrowly tailored.


DESIGNATED (LIMITED) PUBLIC FORUMS


When government may regulate:

If government opens the place to speech, the rules for public forums apply:

  • properly drawn time, place, or manner restrictions are permissible;

  • content-based restrictions get strict scrutiny

Widmar v. Vincent: A university may not exclude religious student groups from using its facilities when other student groups have been granted access to university buildings for speech purposes.

Lamb’s Chapel: A school may not exclude members of the public who engage in religious speech from using school facilities when other public groups have been granted access to school facilities for speech purposes.

Good News Club v. Milford Central School: The Court ruled that a school violated the free speech rights of a private Christian organization by excluding it from meeting after hours in the school building. The Court, following Widmar and Lamb’s Chapel, ruled that this was unconstitutional viewpoint discrimination in a limited public forum, and thus violated the free speech rights of the Good News Club. The Court also opined that permitting the Club to meet on school premises after hours would not establish religion, and so the school had no compelling reason to justify excluding the Club from its designated limited forum.

DESIGNATED (LIMITED) PUBLIC FORUMS

If government closes the place to speech if it wanted to:

Total or selective restriction (including exclusion) of speakers is permissible -- if the restriction is rationally related to the legitimate government interest in maintaining the normal, non-speech use of the public property.

Wagner: If a public school does designate its school building as a public forum, then they cannot choose to have some parts of the community come in, but not other parts.

Total or selective restrictions on speakers may not be based on government dislike of the speech:

They must be justified by some content-neutral reason pertaining to the normal operation of the public place.

NON-PUBLIC FORUMS

Nonpublic forums are government properties that the government can close to all speech activities.


When The Government May Regulate

The government may prohibit or restrict speech in nonpublic forums so long as the regulation is reasonable and viewpoint neutral.

Adderley v. Florida (1966) (1361 text) A state may prohibit protesting on jail grounds. The Court said that in this context the state has the same right as the owner of private property to preserve its property for its intended use. The dissent considered the jailhouse to be a public forum.

Greer v. Spock (1976) (1362 text) Fort Dix is not a traditional public forum, so there is no presumptive right of access to the base for speech purposes. The regulation forbidding political speech was rationally related to the legitimate military objective of keeping military activities free from partisan politics.

Wagner: The government is going to argue that a military base is not one of the traditional places for speech. It is a non-public forum; therefore it can be regulated if it’s rationally related to a legitimate government purpose.

Lehman v. City of Shaker Heights (1974) (1364 text) The government wins here because it didn’t have the majority of justices saying it was a designated public forum. The city has consciously limited access to its transit system advertising space in order to minimize chances of abuse, the appearance of favoritism, and the risk of imposing upon a captive audience.

United States v. Kokinda (1990) (1366 text) The mail must run on time. The only purpose of the sidewalk is for someone to carry mail in or out. The intrusion creates significant interference with Congress’s mandate to ensure the most effective and efficient distribution of the mails. Justice Kennedy concurring said you could use a time, place, and manner regulation to achieve the same purpose.

International Society for Krishna Consciousness, Inc., v. Lee (1992) (1369 text) Government-owned airport terminals are not public forums for speech purposes; therefore, the government may prohibit activities that are inconsistent with the use of its terminals. Wagner: traditionally airports have not been used as a forum for free speech but what about that argument fifty years from now?

Arkansas Educational Television Commission v. Forbes (1998) (1371 text) Public television broadcasters have the right to exclude minor-party candidates from political debates as long as the exclusion is reasonable and not based on the candidate’s views.

Government Properties and Speech


WHAT DETERMINES THE STATUS OF THE FORUM?

Supreme Court has never stated clear criteria for determining whether a place is a public forum, a designated public forum, or a nonpublic forum.

Criteria implicit in the cases – Court considers:

1. The tradition of availability of the place for speech;

2. The extent to which speech is incompatible with the usual functioning of the place;

3. Whether the primary purpose of the place is for speech.

No 1st Am Right of Access to Private Property for Speech Exists

The previously discussed cases each involve claims of a right to use government property for speech purposes.

No right to use private property owned by others for speech exists.

Because it is private property, the Constitution does not apply. (No state action)

Speech In Authoritarian Environments

Another way in which place matters is with regard to certain institutions where Court gives great deference to government to regulate speech

Military; prisons; and schools

Generally only rational basis review given to government regulation of speech in these places.

MILITARY

Parker v. Levy (1974) (1378 text)

Brown v. Glines

PRISONS

General Test: The government may restrict and punish speech of prisoners if the action is reasonably related to a legitimate penological interest.

Thornburgh v. Abbott (1989) (1380 text)

Beard v Banks (2006) (supp)

Shaw v. Murphy (2001) (1385 text),

SCHOOLS

Wagner: More often than not the government’s regulation will be upheld in a school environment because of the need to maintain the authority. Rational basis review is usually given to regulation in these places.

Compare:

W.VA State Board of Education v. Barnette

Tinker v. Des Moines Independent Community School District (1969) (1385 text)

Papish v. Bd of Curators of the Univ. of Mo.

With:

Bethel School Dist.403 v. Eraser (1986) (1388 text)

Hazelwood Sch Dist. v. Kuhlmeier (1988) (1390 text)

SPEECH RIGHTS OF GOVERNMENT EMPLOYEES AT WORK

The government may punish public employees for speech if:

1. It involves matters of public concern; and

2. The state can prove that needs of the government outweigh the speech rights of the employee

Additionally, speech is not protected if made by a government employee while on the job and the speech is part of his or her duties

Garcetti v. Ceballos (2006) (Supp)

Assignment 10 – Freedom of Expression

Freedom of Association; Freedom of the Press

FREEDOM OF ASSOCIATION:

1) Laws Prohibiting & Punishing Membership;

2) Laws Requiring Disclosure of Membership;

3) Compelled Association;

4) Laws Infringing upon Freedom of Association based upon the Expressive Message of the Group

FREEDOM OF PRESS:

1) Freedom of the Press as a Shield;

2) Freedom of the Press as a Sword

FREEDOM OF ASSOCIATION

Freedom of association is a fundamental right protected by the First Amendment.

Laws Prohibiting & Punishing Membership

Laws Requiring Disclosure of Membership

Compelled Association

Laws Infringing upon Freedom of Association based upon the Expressive Message of the Group


Laws Prohibiting or Punishing Membership

Government may punish membership in a group only if it proves that a person:

is actively affiliated with the group,

knew of the group’s illegal objectives, and

had the specific intent to further those illegal objectives.

Laws Requiring Disclosure of Membership

Where disclosure will chill association, the government may require disclosure of membership only if it meets strict scrutiny.

NAACP v. Ala. ex. rel Patterson (1958) (1398 text) A state cannot compel the disclosure of the membership list of a political organization.

Shelton v. Tucker

Laws Concerning Compelled Association (omitted from assigned reading)

Government can require that members of labor union pay union dues, but only to cover collective bargaining activities, not for political activities of the union.

Abood v. Detroit Board of Education (1977) Union dues may not be spent to advance political views opposed by a union member. It violates the employee’s First Amendment right not to be associated with certain causes or ideas.

Government can require bar dues to pay for bar regulatory activities, but not for political activities of bar.

Keller v. California State Bar (1990) Bar association dues may not be spent to advance political views opposed by a member of the bar.

Compare: Mandatory viewpoint neutral student activity fees upheld.

Board of Regents of the Univ. of Wis. v. Southworth (2000) (1401 text) A state university may allocate mandatory student activity fees to political and ideological groups even if some students object to those subsidies. The program, which is viewpoint neutral, is designed to increase the diversity of speech on campus.

Laws Infringing upon Freedom of Association based upon Expressive Message of the Group

Some state and local laws prohibit discrimination by private groups.

Freedom of association protects a right to discriminate only if:

it is intimate association or,

where the discrimination is integral to expressive activity of the group discriminating.


“Infringements on the right to free association may be justified by regulations:

adopted to serve compelling state interests,

unrelated to the suppression of ideas,

that cannot be achieved through means significantly less restrictive of associational freedoms.”

Gender Discrimination by Private Associations

Roberts v. US Jaycees (1984) (1405 text) A state may require private associations to accept women as members. There are two kinds of freedom of association: intimate and expressive.

Freedom of Intimate Expression: Relative smallness of the group, a high degree of selectivity in starting and maintaining the affiliation, and seclusion from others in important aspects of the association (the Jaycees failed to meet this criteria).

Freedom of Expressive Association: Associating for First Amendment purposes, i.e. speech, assembly, and petitioning for redress of grievances. Freedom to associate for expressive purposes presupposes a freedom not to associate.

See also, Bd of Directors of Rotary International v. Rotary Club of Duarte

Freedom of association protects a right to discriminate if the law infringes upon Freedom of Association based upon the expressive message of the group

Hurley v. Irish-American Gay, Lesbian, and Bisexual Group of Boston, (1995) (1409 text)

Boy Scouts of America v. Dale (2000) (1412 text) A state law forcing the Boy Scouts to admit a homosexual as an assistant scoutmaster would violate the Boy Scouts’ First Amendment right of expressive association.

FREEDOM OF THE PRESS

S & S: Under the First Amendment, neither Congress nor a state may pass a law abridging freedom of the press. The main purposes of a free press are to inform the public and to serve as a check on a government (obviously these overlap).

Congress shall pass no law … abridging the freedom of … the press… Const. Am 1

Freedom of the Press as a Shield;

Freedom of the Press as a Sword

Freedom of the Press As a Shield to Protect Press From the Government

Taxes that single out the press are unconstitutional

but government can require press to pay general taxes applicable to all businesses.

Compare:

Grosjean v. American Press Co and Minneapolis Star & Tribune Co. v MN Comm. of Rev (1983) (1418 text)

With

Arkansas Writers’ Project, Inc. v. Ragland,

With

Leathers v. Medlock;

Turner Broadcasting System, Inc. v. FCC, (1994) (not in text)

Constitutional protection of freedom of the press does not entitle the Press to exemptions from general regulatory laws.

Cohen v. Cowles Media Co.(1991) (1423 text) Members of the media have no special immunity from a civil suit against them for breach of a promise not to divulge the name of a source.

See also:

Associated Press v. United States

(no exemption to antitrust statutes)

Associated Press in Citizens Publishing Co. v. United States

(no exemption to antitrust statutes)

Associated Press v. NLRB.

(no exemption to labor laws)

Oklahoma Press Publishing Co. v. Walling

(no exemption to liability under state contract law)

Freedom of the Press Does NOT Protect Secrecy of Reporter’s Sources

Branzburg v. Hayes (1972) (1425 text) Reporters do not have any special privilege that exempts them from testifying before a grand jury. The First Amendment does not guarantee the press a constitutional right of special access to information not available to the public generally.

The Press is not entitled to exemptions from general laws; anyone would have to answer questions from a grand jury, so a reporter must also.

Searches of Newsrooms

the First Amendment does not protect the press from valid searches pursuant to valid warrants.

Zurcher v. Stanford Daily (1978) Members of the media have no special protection from search warrants. Acting pursuant to a valid search warrant, police officers may search the offices of a newspaper for evidence of a crime.

First Amendment not violated by government requirements that the media make broadcast time available to respond to personal attacks as applied to the broadcast media

Red Lion Broadcasting Co. v. FCC (1969) (1432 text) The Court held that Congress and the FCC do not violate the First Amendment by requiring a radio or television station to give reply time to answer personal attacks or political editorials. Focusing on the right of the public to receive information, the Court said that it is permissible for Congress and the FCC to try to ensure fairness in radio and television broadcasting because there were more persons who wanted to broadcast than there were frequencies available. Since “marketplace” was limited, imposing fairness was constitutional.

First Amendment violated by government requirements that the media make newspaper space available to respond to personal attacks as applied to the print media

Miami Herald v. Tornillo (1974) (1436 text) Unlike the broadcast media, a newspaper may not be forced to provide a “right to reply.” Electronic media is different from newspapers because it enjoys a government-granted monopoly through the FCC.

Freedom of the Press As a Sword to Gain Access to Government Places and Papers

The public generally has a right of access to court proceedings, but the Press has NO preferred right of access –

Seattle Times Co .v . Rhinehart

Compare:

Gannett Co. v. DePasquale Neither the public nor the press has s Sixth Amendment right of access to criminal trials; the Sixth Amendment right to a public trial belongs to the defendant.

with

Richmond Newspapers v. Virginia (1980) (1439 text); The Court ruled that the public and the press have a First Amendment right to attend criminal trials.

Globe Newspaper Co. v. Superior Court (1982) A state may not close every proceeding involving testimony by a minor victim of assault.

Press-Enterprise Co. v. Superior Court (1984) There is a presumptive First Amendment right of the public and the press to attend voir dire of potential jurors in a criminal case.

The public has no right of access to prison inmates and facilities

The press has NO special right of access.

Pell v. Procunier (1973) (1445 text) Reporters have no special right of access to prisons or inmates whom they wish to interview beyond that afforded to the general public.

Houchins v. KQED (1978) (1449 text) Reporters have no special right of access to jails than does the general public.

Constitutional Law II

Assignment 11


FREEDOM OF RELIGIOUS CONSCIENCE

Fundamental Right to Free Exercise of Religion and Freedom of Religious Conscience

“Congress shall make no law respecting an establishment of religion (Establishment Clause), or prohibiting the free exercise thereof (Free Exercise Clause)”.

U.S. Const. Amend 1

Wagner: Understand the placement of those words – they are in the First Amendment. Historically, there was state persecution against the church and state-run churches.

The underlying jurisprudential battle between the justices is between an idea that there is a standard we can look to opposed to a morally subjective analysis in looking at the Constitution – one that evolves or one that is reflective of the document.

Think of the federal government passing a law that infringes on the free exercise of religion. The question then becomes, is there some other part of the Constitution that will limit the government’s right from doing that. In that situation you should think about the Free Exercise or Establishment Clauses. This is a fundamental right. If you don’t have this and free speech, you don’t have a functional democracy.

This applies to the states also. We use the First Amendment and incorporate that into the Fourteenth Amendment (incorporating a fundamental right to apply to the states).

Free Exercise Clause situation: an individual who is trying to exercise a right and a government trying to burden or substantially infringe on that right.

Different colonies had different state religions. The framers were concerned about a federal religion and that could preempt a state religion.

The Lemon Test

The Establishment Clause violates the Lemon test if the government’s primary purpose is to advance religion or it has a principle effect of aiding or inhibiting religion or there is excessive government entanglement.

With a definition like that, you are just asking for a mess anytime the government does anything that remotely can be connected to religion whether it accommodates religion or whether it doesn’t accommodate religion.

Religious Test Prohibition

“ “The Senators and Representatives before mentioned, and the Members of the several State Legislators, and all executive and judicial Officers, both of the United States and of the several States, shall be bound by Oath or Affirmation, to support this Constitution but no religious Test shall ever be required as a Qualification to any Office or public Trust under the United States.”

U.S. Const. Art. VI Cl. 3

Wagner: You cannot stop somebody running for office because of what he or she believes.

Review: Defining Religion

Requirement for Sincerely Held Beliefs

United States v. Ballard (1944) (1459 text) Jury Determination of Religious Beliefs – A jury may not question the truth or falsity of religious beliefs. The Court ruled that a jury may not be given the question of whether particular religious beliefs are true. The defendants in this case, who had been charged with mail fraud, claimed to be divine messengers who had talked with Jesus, and who had the power to cure incurable diseases. The Court held that the First Amendment guarantees the protection of a person’s chosen form of religion regardless of how incomprehensible those views may be to others.

Wagner: You cannot make up stuff. You have to have a sincerely held belief. Proving what someone believes could create huge problems if we decide that bogus could be defined in court every time.

When does a man’s manifestationable belief go to the point where government can regulate?

The Relevance of Religious Doctrines and Shared Beliefs

Frazee v. Illinois Employment Security Department An individual who asserted that he was a Christian refused a temporary position offered him by an employment agency on the grounds that he could not work on "the Lord's day." The position would have required him to work on Sunday. He was not a member of an established religious sect or church and did not claim that his refusal to work resulted from a tenet, belief, or teaching of an established religious body. He then applied for unemployment compensation benefits to the Illinois Department of Employment Security, which denied his claim both initially and on appeal to its review board. The Department's determination was affirmed by the Circuit Court of the Tenth Judicial Circuit of Illinois, Peoria County. On further appeal, the Appellate Court of Illinois, Third District, while not questioning the sincerity of the individual in his "personal professed religious belief," affirmed, holding that the free exercise clause of the Federal Constitution's First Amendment had not been violated (159 Ill App 3d 474, 111 Ill Dec 400, 512 NE2d 789). The Illinois Supreme Court denied leave to appeal.


On appeal, the United States Supreme Court reversed and remanded. In an opinion by White, J., expressing the unanimous view of the court, it was held that the denial of unemployment compensation benefits to the individual violated the free exercise clause, since the state presented no interest which was sufficiently compelling to override the individual's free exercise of his sincerely held religious belief.

Thomas v. Review Board of the Indiana Employment Security Division (1981) (individual can claim a religious belief even though it is inconsistent with the doctrines of his or her religion) The Court reversed a state court’s denial of unemployment compensation benefits to a JW who quit his job because his religious beliefs would not allow him to participate in the production of armaments. He had been transferred to a department that made tank turrets. Relying on Sherbert, the Court found that the denial of benefits to Thomas substantially burdened the free exercise of his religion, and was not supported by any compelling government interest.

ASK: whether the individual holds a sincere religious belief.

FREE EXERCISE CLAUSE

Free Exercise Clause - Religious Beliefs v. Actions

Government may not compel or punish religious beliefs;

people may think and believe what they want.

The free exercise clause, however, does not provide absolute protection for religiously motivated conduct.

WHEN is the Free Exercise Clause most likely to be invoked?

1. When the government prohibits behavior that a person’s religion requires; If you see state action (legislation or some other type of government action) that prohibits behavior that a person’s sincerely held religious belief requires, then you are going to pull out the First Amendment.

2. When the government requires conduct that a person’s religion prohibits; When you see the government requiring conduct that a person’s religion prohibits, again the First Amendment Free Exercise Clause can be something you bring up that limits the government action.

3. When individuals claim that laws burden or make more difficult religious observances; (Balance) When an individual claims that a state action or a government law burdens or makes more difficult a religious observance, that is another time that you are going to bring up the First Amendment.

The difference in #3 is just a burden as opposed to prohibiting.

Freedom of Religious Conscience
Free Exercise Clause – Underlying question for SCT

Should freedom of religious conscience be protected through exemptions from general laws?

Some think strict scrutiny of government action is necessary when government significantly burdens religion.

Notwithstanding the express language of the Free Exercise Clause, others believe that there should not be religious exemptions from general laws.

Wagner: A general law gets passed so you have state action. That law places a burden on someone’s religion. The question becomes then, is the free exercise clause violated because of that burden? Can I put up the free exercise clause and say it is a limit on the state’s ability to regulate me.

This is a fundamental right; therefore, apply strict scrutiny. There have been some situations however, when the government did not apply strict scrutiny.

For Exam:

Wagner: Some justices have an understanding that strict scrutiny is essential when the government is significantly burdening a fundamental of the free exercise of religious conscience. That clearly was the view in the early cases.

Notwithstanding the language that Congress shall pass no law…you’ve got others on the Court that say there should not be an exemption from a general law that applies to everybody even if it burdens somebody’s free exercise of religious conscience. It does not limit the government’s ability to regulate because they are doing it in a general way.

Freedom of Religious Conscience
Free Exercise Clause – Pre-1960

Challenges to Law Regulating or Burdening Religious Conduct

Court distinguished between beliefs and action in violation of social duties or subversive of good order

The free exercise limited government regulation of the former, but not the latter.

Court on this basis rejected claim that clause requires an exemption from an otherwise valid criminal law.

Reynolds v. United States (1878) The Court upheld a federal law prohibiting polygamy as applied to a Mormon whose religion required polygamy. The Court said that Congress could not regulate religious beliefs but could reach actions that were violative of social duties or subversive of good order. This law was applied generally and was challenged solely on Free Exercise grounds.

Court applied the free exercise clause to the states in 1940 (Cantwell v. Connecticut)

Before Cantwell, Court protected religious freedom under other constitutional provisions, especially the due process clause of the 14th Amendment.

Pierce v. Society of Sisters (1925) The Court held law infringed on the right of parents to control the upbringing of their children and thus impermissibly violated the liberty guaranteed under the due process clause of the Fourteenth Amendment.

Supreme Court’s initial explicit protection of free exercise of religion occurred in a series of cases holding unconstitutional various laws restricting religious groups from soliciting funds.

Cantwell v. Connecticut (1940) Licensing system for religious solicitations violates both free exercise and free speech clauses of 1st Amendment. JW’s were soliciting as a manner of their manifestation of their sincerely held religious belief and the Court struck down their conviction.

Freedom of Religious Conscience
Free Exercise Clause – 1960-1990 Strict Scrutiny

In evaluating government laws burdening religious freedom strict scrutiny was the appropriate test.

Striking Down Laws Examples:

Sherbert v. Verner (1963) (1473 text) A state cannot deny benefits to a person who refuses to work on her Sabbath (Saturday, Seventh Day Adventist). The state’s refusal to grant Sherbert an exemption substantially burdened her exercise of religion and that the state had no compelling reason to deny the exemption.

Wagner: What choice does she have? I’m either going to follow my sincerely held religious belief or I’m going to eat and have shelter.

Thomas v. Review Board (1981) A state cannot deny unemployment compensation benefits to a person who has religious reservations about making weapons. Relying on Sherbert, Thomas’s free exercise of his religion was substantially burdened and was not supported by any compelling government interest.

Hobbie v. Unemployment Appeals Commission of Florida (1987) A state cannot deny benefits to a worker whose religious beliefs developed after she had been on the job.

Frazee v. Illinois Department of Income Security (1989) Denial of unemployment compensation benefits to nondenominational Christian who refused to work on Sunday, based on religious beliefs, violates the First Amendment's free exercise clause.

Wisconsin v. Yoder (1972) A state cannot apply a mandatory education law to the Old Order Amish.

Wagner: There are two ways that this government action can be eliminated (Hybrid or Two different jurisprudential avenues):

1. There is a fundamental substantive due process right of a parent to control the upbringing of their children.

2. But because the Amish are not willing to send their kids to high school because of their sincerely held religions belief, this would violate the Free Exercise Clause.

It’s not because the teachers are inept, but because of a sincerely held religious belief.

Some justices would say that those are two separate jurisprudential doctrines of winning your case by limiting state action under strict scrutiny.

Other justices are saying (as in the Smith case) that it’s a hybrid and you do not apply strict scrutiny to laws that have a general applicability. They distinguish Yoder by saying that it wasn’t just a First Amendment case – it was a hybrid.

Upholding Laws Examples:

Braunfield v. Braun (1961) Upholding Sunday closing law – compelling government interest in uniform day of rest (Sunday) as a state could not be expected to recognize exceptions (Saturdays for Jews) for all whose religions required a day of rest other than Sunday. Here there was no direct burden on free exercise rights.

United States v. Lee (1982) The federal government can apply the social security tax system to those whose religion forbids participation (Amish). Mandatory participation is the only way to achieve the fiscal integrity of the social security system.

Bob Jones University v. United States (1983) The federal government can deny tax-exempt status to schools that discriminate based on race – even if the discrimination was the result of a sincerely held religious belief. The government interest in eliminating racial discrimination outweighed any incidental impact on the Free Exercise rights of the schools, their students, and the parents of those students.

Bowen v. Roy (1986) Forcing Native Americans to submit and use an identification number (social security number) does not violate the Free Exercise Clause. This is only an incidental burden and is clearly outweighed by the interest in an efficient welfare system.

Freedom of Religious Conscience
Free Exercise Clause – Post 1990

Court expressly rejected use of strict scrutiny for challenges to neutral laws of general applicability burdening religion (use rational basis review)

Sacramental Use of Peyote

Employment Division v. Smith (1990) (1464 text) The Court upheld a state’s power to criminalize the use of peyote (a state drug law that applies to everyone), even when used by Native Americans as an essential part of their religion. Rule: A generally applicable law which has only an incidental effect of burdening religion does not offend the Free Exercise Clause. No heightened scrutiny is called for in such a case.

Wagner: Smith shook the religious communities within the country. The type of government interest rejected by Smith was laws that apply to everybody. If the government passes a general law that applies to everybody, then you don’t get to raise the First Amendment even if it substantially interferes with your free exercise of a sincerely held religious belief – it is no longer a limit on government action if it is rationally related to a legitimate government purpose. It is no longer a fundamental right.

Smith is distinguished from the other cases because they were hybrids and strict scrutiny was applied. There were other fundamental rights involved in those cases. Smith creates an environment at the state level that says if a state passes a law, as long as it’s generally applicable to everybody and neutral, all it has to do is meet the rationally related test.

Sale of Alcohol on Weekends Hypo: For those taking the sacrament with wine – before Smith; exemption – no compelling reason and no least restrictive means. After Smith; no exemption – it applies to everyone and the government could argue that there is a legitimate government purpose, i.e. drunk driving.

Activities of Specific Religious Groups

A law that is not neutral or not of general applicability “must be justified by a compelling interest and must be narrowly tailored to advance that interest.”

Church of the Lukimi Babalu Aye, Inc. v. Hialeah (1993) (1478 text) A state cannot selectively prohibit activity engaged in by a specific religious group, while allowing other religious groups (or slaughterhouses) to engage in that activity. In addition to the free exercise argument of the Santeria religion, it is also arguable that the government is establishing religion by allowing other religious groups to maintain slaughterhouses.

S & S: If a law has exceptions or exemptions built into it, strict scrutiny will apply, even though the law is challenged solely on Free Exercise grounds.

Freedom of Religious Conscience
Free Exercise Clause – Congress Responds – RFRA

Congress adopted the Religious Freedom Restoration Act of 1993 to negate the Smith test and require strict scrutiny for free exercise clause claims. Congress wants to restore strict scrutiny to government laws that infringe upon someone’s free exercise of religious conscience.

“Government shall not substantially burden a person’s exercise of religion even if the burden results from a rule of general applicability, except. . . [g]overnment may substantially burden a person’s exercise of religion only if it demonstrates that application of the burden to the person (1) is in furtherance of a compelling governmental interest; (2) is the least restrictive means of furthering that compelling government interest.”

Freedom of Religious Conscience
Free Exercise Clause – City of Boerne v. Flores

Supreme Court declared the RFRA unconstitutional as to states…

Court ruled that Congress exceeded the scope of its power under Section 5 of the 14th Amendment in enacting RFRA.

Con Law I – Review

Scope of Congress' power under section 5 of the 14th Amendment (not a limit but a power source – it empowers Congress to make laws to enforce the 14th Amendment).

Narrow View Example:

City of Boerne v. Flores (1997) (216 text)

Free Exercise case: City of Boerne v. Flores, 521 U.S. 507 (1997), was a Supreme Court case concerning the scope of Congress's enforcement power under the fifth section of the Fourteenth Amendment.

The case arose when the Catholic Archbishop of San Antonio applied for a building permit to enlarge a church in Boerne, Texas. Local zoning authorities denied the permit, relying on an ordinance governing building preservation in a historic district which, they argued, included the church. The Archbishop brought a lawsuit challenging the permit denial under the Religious Freedom Restoration Act of 1993 (RFRA, pronounced "rifra").

Before this case Congress could tell a state court that when they have a free exercise argument, they must use strict scrutiny, not rational basis scrutiny – they dictate to state courts what level of review they have to use.

The Court, in an opinion by Justice Anthony Kennedy, struck down RFRA as an unconstitutional use of Congress's enforcement powers. Now Congress can’t go telling state courts how to function.

Congress, under § 5 of the 14th Amendment, has authority only to enact laws that prevent or remedy violation of rights already recognized by the Supreme Court; and such laws must be narrowly tailored, i.e. “[t]here must be a congruence and proportionality between the injury to be prevented or remedied and the means adopted to that end.”

The Supreme Court said RFRA failed these requirements and declared it unconstitutional.

Boerne invalidated RFRA as to state and local governments. Its reasoning does not apply to the constitutionality of the law as applied to the federal government.

Wagner: I wonder if RFRA is still good law with regard to federal laws that are passed by the US Congress? Hmm…

Congressional power source to regulate state and local governments here was Section 5 of the 14th Amendment but the 14th Amendment does not apply to the federal government, thus, RFRA not unconstitutional as to federal government.

The Supreme Court approves a rigorous application of strict scrutiny of federal actions covered under RFRA.

Gonzales v. O Centro Espiritu Beneficente Uniao do Vegetal (2006) (232 Supp) The Court rejected the government’s argument that it has a compelling interest in the uniform application of the Controlled Substances Act (claimants take a hallucinogen for religious purposes), asserting that RFRA’s strict scrutiny test contemplates that the government must demonstrate that the compelling interest test be applied to a specific person – the specific claimant whose sincere exercise of religion is being substantially burdened. “The government’s mere invocation of the general characteristics of Schedule I substances, as set forth in the CSA, cannot carry the day.”

Freedom of Religious Conscience
Free Exercise Clause – Congress Responds – Religious Land Use and Institutionalized Persons Act (RLUIPA) of 2000

Congress adopted the Religious Land Use and Institutionalized Persons Act in 2000

“No government shall impose a substantial burden on the religious exercise of a person residing in or confined to an institution unless the burden furthers a compelling government interest and does so by the least restrictive means.”

The law requires government to meet strict scrutiny when it significantly burdens religion in two areas:

1. Cases involving land use decisions; and

2. Cases involving institutionalized persons.

Cutter v. Wilkinson (2005) (229 supp) The Court acknowledged that the federal application of RFRA had not been decided. The Court acknowledges that RLUIPA may devolve one day into an unlawful fostering of religion but on its face, RLUIPA does not exceed the limits of permissible government accommodation of religious practices.

RLUIPA protects institutionalized persons who are unable freely to attend their religious needs and are therefore dependent on the government’s permission and accommodation for exercise of their religion.

Freedom of Religious Conscience
Free Exercise Clause – Summary: Current Law

For Exam Use This Test:

I. For state and local governments:

A. Smith controls – one cannot use Free Exercise Clause to challenge neutral laws of general applicability.

EXCEPT a state or local decision that places a burden on religion concerning land use and an institutionalized person, you have to meet strict scrutiny (RLUIPA – in that case, apply strict scrutiny).

II. For federal government:

A. RFRA requires that federal actions burdening religion meet strict scrutiny.

Freedom of Religious Conscience
Free Exercise Clause – Government Aid for Religious Education

State does not violate establishment clause by permitting students receiving scholarship assistance to study for the ministry

Aid for Handicapped Students

Witters v. Washington Dept. of Services for the Blind (1986) The Court relied on the fact that this was a generally applicable aid program and any money that ultimately went to religious institutions did so as a result of the independent choices of private recipients.

State does not violate the Free Exercise Clause by allowing students to receive scholarship assistance only if they pursue secular, (but not religious) studies.

Locke v. Davey (2004) (1482 text) The state of Washington gave scholarships to everyone unless you studied devotional theology. The Court found no violation of the religion clauses of the First Amendment because no animus to religion was found. The state isn’t denying a student’s pursuit of a degree in devotional theology; it’s only saying that it is not going to pay for it. The government can spend money how it wants to.

Wagner: Based on other things you’ve learned, for the “A” answer, under the doctrine of content-based speech restriction (taking it to strict scrutiny), the state created a metaphorical public forum that an educational institution is one of the cornerstones of free speech and place of ideas. The state is going to fund all kinds of speech except for devotional theology. A designated public forum has to be content neutral. Here, the restriction is content based; therefore must meet strict scrutiny.

Constitutional Law II
Assignments 12-13
Freedom of Religious Conscience

The Establishment Clause

Freedom of Religious Conscience

“Congress shall make no law respecting an establishment of religion, or prohibiting the free exercise thereof.”

U.S. Const. Amend 1

Freedom of Religious Conscience
Establishment Clause – Three Interpretive Views

Strict Separation View

Neutrality Theory

Accommodation Approach

Freedom of Religious Conscience
Establishment Clause - Govt Discrimination Among Religions

Government Discrimination Among Religions violates the Establishment Clause unless it survives strict scrutiny review

Larson v. Valente (1982)

(1497 in text – not assigned, but discussed in class)

If govt is not discriminating among religious groups, Court uses other tests (e.g., Lemon test)

Freedom of Religious Conscience
Establishment Clause - The Lemon Test

If a law is not discriminatory, the Supreme Court sometimes applies the 3-part Lemon test:

First, the statute must have a secular legislative purpose;

Second, its principal or primary effect must be one that neither advances nor inhibits religion;

Third, the statute must not foster an excessive government entanglement with religion.

Government violates the Establishment Clause if it violates any prong - Lemon v. Kurtzman (1971) (1498 text)

Freedom of Religious Conscience
Establishment Clause - Religious Speech and the 1st Am

When is exclusion of religion a violation of speech clause?

Where the government chooses to restrict private religious speech on government property or with government funds because of a desire to avoid violating the establishment clause à the Supreme Court consistently has held that excluding such religious speech violates the First Amendment protection of freedom of speech

because it is an impermissible content-based restriction of expression.

Freedom of Religious Conscience
Establishment Clause - Religious Speech and the 1st Am
When is exclusion of religion a violation of speech clause?

Religious Group Access to School Facilities - General Rule:

Government sponsored religious activities in public school classrooms are not allowed.

However, if govt makes its facilities available to students and community groups, govt cannot discriminate against religion.

Widmar v. Vincent,

Bd of Ed of Westside Community Schools v. Mergens,

Lamb’s Chapel,

Good News Club

Freedom of Religious Conscience
Establishment Clause - Religious Speech and the 1st Am

When is exclusion of religion a violation of speech clause?

Student Religious Group Receipt of Government Funds

Although government has wide discretion when it chooses to allocate scarce financial resources,

viewpoint-based restrictions are NOT proper when the government expends funds to encourage a diversity of views from private speakers (as opposed to when govt itself speaks or subsidizes transmittal of a message it favors).

Rosenberger v. Rector and Visitors of the Univ of Va (1995) (1504 text)

Freedom of Religious Conscience
Establishment Clause - Religious Speech and the 1st Am
When is exclusion of religion a violation of speech clause?

Student-delivered Prayer at Football Games

Some attempts to use 1st Am speech analysis to allow more religious presence in government activities have failed

Santa Fe Independent School District v.Doe (2000) (1508 text),

Freedom of Religious Conscience
Establishment Clause - Religious Speech and the 1st Am

When is exclusion of religion a violation of speech clause?

Be aware that in the future some traditional establishment clause issues might be re-conceptualized by the Court as involving government content-based discrimination against speech…..

Freedom of Religious Conscience
Establishment Clause - When Can Religion Become a Part of Government Activities?

Release Time from classes to receive religious instruction

McCollum v. Board of Education

Zorach v. Clauson

School Prayers and Bible Reading

Engel v. Vitate (1962) (1514 text)

Abington School District v. Schempp

Wallace v. Jaffree Lee v. Weisman (1992) (1517 text)

Santa Fe Independent School District v. Doe

Curricular Decisions

Epperson v. Arkansas

Edwards v. Aguillard

Freedom of Religious Conscience
Establishment Clause - When Can Religion Become a Part of Government Activities?

Religious Symbols on Government Property

Lynch v. Donnelly

County of Allegheny v. American Civil Liberties Union

McCreary County, Kentucky v. ACLU of Kentucky (2005) (233 supp)

Van Orden v. Perry (2005) (249 supp)

Legislative Chaplains

Marsh v. Chambers (1983) (1525 text)

Freedom of Religious Conscience
Establishment Clause - When Can Govt Give Aid to Religion?

Four major areas where the Court has considered government aid to religion:

tax exemptions for religious institutions,

assistance to parochial elementary and secondary schools,

aid to religious colleges and universities,

and assistance to religious institutions other than schools.

Constitutional Law II
Assignments 12-13


Freedom of Religious Conscience

The Establishment Clause

“Congress shall make no law respecting an establishment of religion, or prohibiting the free exercise thereof.”

U.S. Const. Amend 1

Freedom of Religious Conscience
Establishment Clause – Three Interpretive Views

Wagner: The theory a particular justice chooses to use when analyzing an establishment clause case will determine the result. In every establishment clause case the following theories are present:

Strict Separation View

Neutrality Theory

Equality Approach

Strict Separation View

Strict Separation: The government should be secular and there should be a wall preventing religious view points. If you are a secular nation (like France) and we are separating church and state, the purpose of strict separation is to keep out religion from political decision making.

Strict separationists seek a secular nation, not a pluralistic nation where there is policy making and everybody can participate in the policy making as long as their basis for their position is a secular one and not a religion based one.

Strict separationists are trying to prevent a theocracy where someone is imposing religious viewpoints or have a religious influence in decision making of the government. This approach tends to exclude religious viewpoints. This approach favors the Lemon test.

Neutrality Theory

Neutrality/Symbolic Endorsement: The government cannot favor one religion over another. A justice will look to see if neutrality is there. The people who support this theory think this is the right test is because if you show favoritism over secular folks, then the secularists will feel excluded. Or, if you show favoritism to one religion over another religion then the people in the excluded religion will feel less in their community.

Critics of this approach say that the reverse is also true – when censoring a religious viewpoint are you really being neutral? If you make a secularist feel outside, are you being neutral? When you censor a secularist viewpoint, are you really being neutral? Are you sending a clear message to those with sacred viewpoints that they are less than full members of the American community?

Supporters of this theory say that this is a good way to tell if the government is neutral or whether they are favoring religion. But how can something that does not favor religion be neutral? That’s the tougher question that the justices don’t answer. This approach favors the Lemon test.

Equality Theory

Accommodation/Equality/No Coercion Approach: The government is required to treat sacred beliefs and groups that have sacred beliefs equally with groups that have secular or nonreligious beliefs. Under this approach a government action would only violate the establishment clause if it establishes a church or government religion.

Under this approach the Establishment Clause would be violated if the government action attempts to coerce participation in a religion or favors one religion over another. A justice will discuss a lack of coercion in his or her opinion. This approach allows for more religious freedom. This approach does not like the Lemon test.

Understanding these theories is the only way that you can understand (explain the inconsistencies in) the cases.

Freedom of Religious Conscience
Establishment Clause - Government Discrimination Among Religions

Government Discrimination among Religions violates the Establishment Clause unless it survives strict scrutiny review

The Larson test

Larson v. Valente (1982) (1497 in text – not assigned, but discussed in class) The Larson test is used when the government prefers one religion to another. If that happens, the Court uses a compelling interest test to analyze the law. Strict scrutiny is called for because preferring one religion to another goes to the heart of Establishment Clause concerns.

If the government is not discriminating among religious groups, Court uses other tests (e.g., Lemon test)

Freedom of Religious Conscience
Establishment Clause - The Lemon Test

The Lemon Test

If a law is not discriminatory, the Supreme Court sometimes applies the 3-part Lemon test:

First, the statute must have a secular legislative purpose;

Second, its principal or primary effect must be one that neither advances nor inhibits religion; It has to have a secular effect. Is what the government is doing symbolically endorsing religion? If it is, government action struck down – must meet all three prongs.

Third, the statute must not foster an excessive government entanglement with religion. An excessive government entanglement is a law that requires a comprehensive, discriminating, and continuing state surveillance. In other words, if the government has to keep checking on you.

If there is no discrimination among religious groups, then you apply the Lemon test.

Second Prong Examples:

Advancing Religion Hypo: No person required to work on his or her Sabbath. The government action will be struck down because its primary or principal effect advances religion as opposed to an incidental effect.

Title VII Exemption for Religious Groups: Religious groups may discriminate who the leader or president is. An Establishment Clause challenge will be defeated as there is a government and secular interest in keeping government away from religious organizations so they can carry out their own religious mission. It allows a church to advance religion which is its purpose.

Third Prong Example:

Hypo: If a state passes a law that the state will pay the teachers’ salaries in parochial religious schools as long as the teachers are teaching secular subjects like math as well. The law will be struck down because the state would have to keep checking on them. See case below.

Government violates the Establishment Clause if it violates any prong

Lemon v. Kurtzman (1971) (1498 text) The Court invalidated state statutes which supplemented salaries of teachers in parochial schools. Such supplements would result in excessive entanglement between government and religion because the religious school teachers would have to be monitored to make sure they did not sneak religion into the secular subjects that the government was paying them to teach.

Wagner: A justice that is hostile to an Establishment Clause argument may be amenable to a Free Speech argument instead. Good lawyering takes Establishment Clause cases and rephrases the issue as a designated public forum for all speech except for “sacred” speech or religious speech and therefore creating a content-based regulation.

Why government will restrict religious speech on government property, even though they know it’s a public forum, is because the government believes that if it allows it, somebody will challenge it as violating the Establishment Clause.

Freedom of Religious Conscience
Establishment Clause - Religious Speech and the First Amendment

When is the exclusion of religion a violation of speech clause?

Where the government chooses to restrict private religious speech on government property or with government funds because of a desire to avoid violating the establishment clause à the Supreme Court consistently has held that excluding such religious speech violates the First Amendment protection of freedom of speech because it is an impermissible content-based restriction of expression.

Religious Group Access to School Facilities - General Rule:

Government sponsored religious activities in public school classrooms are not allowed.

However, if the government makes its facilities available to students and community groups, the government cannot discriminate against religion.

Widmar v. Vincent (1981) A state university may not exclude a religious student group from a designated public forum.

Board of Education of Westside Community Schools v. Mergens (1990) Public schools may not exclude a religious student group from a designated public forum.

Lamb’s Chapel v. Center Moriches (1993) A school may not exclude religious speakers from a designated public forum open to the public. This would be a content based restriction.

Good News Club v. Milford Central School (2001) P 276 A school may not exclude members of the public who engage in religious speech from using the school facilities when other public groups have been granted access to school facilities for speech purposes.

Student Religious Group Receipt of Government Funds

Although government has wide discretion when it chooses to allocate scarce financial resources, viewpoint-based restrictions are NOT proper when the government expends funds to encourage a diversity of views from private speakers (as opposed to when the government itself speaks or subsidizes transmittal of a message it favors).

Rosenberger v. Rector and Visitors of the Univ. of Virginia (1995) (1504 text) A school may not deny funding to a religious student publication if it funds other student journals. A university does not establish religion by granting access to its facilities to a wide spectrum of student groups, even if some of them are religious. A limited forum enhances diversity. Wagner: the school was taking student money and distributing it to different groups to perpetuate their ideas except they discriminated against to Christian groups. The Court strikes it down as unconstitutional. The theory: We have a public forum that discriminates on viewpoint that survives the neutrality theory.

Student-Delivered Prayer At Football Games

Some attempts to use the First Amendment speech analysis to allow more religious presence in government activities have failed

Santa Fe Independent School District v. Doe (2000) (1508 text) It is unconstitutional for a public school district to allow student-initiated, student-led prayers at high school football games.

Wagner: The majority strikes this down because there is no created designated public forum here; therefore, not discriminating based on religious speech.

Be aware that in the future some traditional establishment clause issues might be re-conceptualized by the Court as involving government content-based discrimination against speech….

Freedom of Religious Conscience
Establishment Clause - When Can Religion Become a Part of Government Activities?

Release Time from Classes to Receive Religious Instruction

McCollum v. Board of Education (1948) Religious teachers cannot hold classes in a public school building where students who were not excused remain in the classroom.

Zorach v. Clauson (1952) Public school students can leave school to go to religious education programs conducted in religious centers.

  • Religious instruction in public school buildings violates the Establishment Clause because it has the effect of endorsing religion and it creates a symbolic union between church and state.

School Prayers and Bible Reading

Engel v. Vitale (1962) (1514 text) A state cannot require that an official school prayer be said in public school classrooms. The Court invalidated the practice of a school board requiring a non-denominational prayer to be said in public school each day. The fact that the prayer was written by the New York Board of Regents was tantamount to an establishment of an official state religion.

Abington School District v. Schempp

Wallace v. Jaffree (1985) States may not require a minute of silence in public schools for meditation or voluntary prayer if the sole purpose is to endorse religion. The Court invalidated an Alabama law which authorized a minute of silence in public schools for meditation or voluntary prayer. This law ran afoul of the purpose prong of Lemon because the state’s sole purpose was to endorse religion.

Lee v. Weisman (1992) (1517 text) Clergy cannot offer the invocation and benediction at graduation ceremonies for junior and senior high school. The Court invalidated the practice of public school principals inviting members of the clergy to offer invocation and benediction prayers at junior high and high school graduations. The principals also provided the clergy with a pamphlet containing guidelines on the content of the prayers to be given. The Court ruled that government involvement with religious activity was so pervasive as to create a state-sponsored and state-directed religious exercise in the public schools. The Court alluded to the subtle coercion that exists in such a setting and said that a student attending graduation could not avoid the fact or appearance of participation in the religious exercise.

Santa Fe Independent School District v. Doe, (See Above)

Curricular Decisions and Teaching of Evolution

Epperson v. Arkansas (1968) A school cannot prohibit the teaching of evolution in public schools. The Court said the law was purely motivated by the religious purpose of advancing fundamentalist Christian beliefs.

Edwards v. Aguillard (1987) State law cannot require the teaching of creation science whenever evolution is taught. The Court ruled that the law had no secular purpose and rejected the state’s argument that it was trying to advance academic freedom by requiring a balanced treatment of the beginning of humankind.

Religious Symbols During the Holiday Season

Lynch v. Donnelly A Christmas holiday display will be deemed constitutional if it is secularized by the inclusion of non-religious symbols.

County of Allegheny v. American Civil Liberties Union A Christmas holiday display will be deemed unconstitutional if it is composed solely of religious symbols.

Ten Commandments Outside the School Context

  • The Court has been inconsistent in ruling on Ten Commandments displays on government property other than schools.

McCreary County, Kentucky v. ACLU of Kentucky (2005) (233 supp)(314-315 S & S) To determine whether a reasonable observer would conclude the government had no secular purpose underlying its action, the Court focused on the history, context, and evolution of the displays. The Court reaffirmed the first prong of the Lemon test. The Court found a secular purpose lacking in all three versions of the displays. The last modification to the display was only a guise to make it look like they were being secular by adding the Bill of Rights, the Magna Carta, the words to the National Anthem, etc.

Wagner: If you want it to be constitutional, put a lot of “stuff” around it (See P 308 S & S) and maybe they won’t notice that your display has a symbolic endorsement or that you are intending to not be neutral. The ACLU is suing because by putting the Ten Commandments on public property, the state is establishing a religion and they shouldn’t.

There are parts of the federal code that merge with religion.

The reasoning behind not allowing a symbolic endorsement is excluding or the appearance of excluding. The majority is scrutinizing the government’s purpose. We cannot close our eyes to the fact that the state flipped the display around three times. The question becomes is this is the correct analysis to use in interpreting the establishment clause or the wrong analysis?

Under the equality approach you treat the purposes the same. Under the neutrality approach, we look to see if the display endorses religion.

The dissent uses an entirely different theory to interpret the Establishment Clause. The bigger battle is how do we interpret the Establishment Clause?

Van Orden v. Perry (2005) (249 supp)(314 S & S) The Court ruled that a display of a monument inscribed with the Ten Commandments on the grounds of the Texas State Capitol does not violate the Establishment Clause. The monument stands among 21 historical markers and 17 monuments and has stood there since 1961. The Court rejected the Lemon test as not useful for dealing with this sort of passive monument on the Capitol grounds. Instead the Court focused on the monument’s nature and the Nation’s history. While the Ten Commandments are undeniably religious, they also have historical meaning. When considering all the monuments and markers together, what Texas has done is treated them as representing several strands in the state’s political and legal history.

Justice Breyer, going through the neutrality theory, finds a secular purpose (historical relationship) and emphasized that the monument went unchallenged for 40 years. Wagner: The cynical answer is that Breyer switched positions

ATTACK STRATEGY

Look to see what the government is doing. The first thing you do even before you get to the Lemon test is check and see if the government is discriminating among religions – is it treating one religion differently than another religion? That type of discrimination indicates that a fundamental right is being violated and strict scrutiny will apply. The government will need a compelling reason why it is treating the two groups differently and will need to use the least restrictive means to accomplish the government interest.

If the government action is not discriminating among religious groups, then you apply the Lemon test (at least for now).

Hypo: If you are a charitable organization, you have to record all the monies you receive, but the law exempted certain types of religious groups that received 50% of income from its members. After applying strict scrutiny the law was struck down because some churches have members and others don’t.

Legislative Sessions

Marsh v. Chambers (1983) (1525 text) The Court upheld the practice of a paid chaplain opening each session of the Nebraska legislature with a prayer. The Court did not rely on Lemon but used a historical analysis, focusing on the fact that when Congress passed the First Amendment, a member of the clergy opened the session with a prayer.

Freedom of Religious Conscience
Establishment Clause - When Can the Government Give Aid to Religion?

Four major areas where the Court has considered government aid to religion:

Tax exemptions for religious institutions;

Assistance to parochial elementary and secondary schools;

Aid to religious colleges and universities; and

Assistance to religious institutions other than schools.

Mitchell v. Helms (2000) (text 1527) (P 313 S & S) The Court upheld a federal school aid program under which the federal government distributes funds to state and local government agencies, which in turn lend educational materials and equipment to public and private (including religious) schools.

Public School Voucher Program

Zelman v. Simmons-Harris (2002) A state school voucher program that offers tuition aid to parents to send their children to secular or religious private schools passes Establishment Clause muster because it is neutral with respect to religion, and any aid flowing to religious institutions is the result of private, independent choices of parents of school children.